Site Loader

Содержание

Вопрос: Вокруг каких зарядов — неподвижных или движущихся — существует электрическое поле, вокруг каких — магнитное поле? Ответ на вопрос – iq2u

Точные науки Физика

Ответ:

Электрическое поле существует вокруг всех зарядов, магнитное — вокруг движущихся.

Проверь себя, пройди тесты онлайн

ОГЭ 2019. Физика. Вариант 26

Статистика теста

0–25%

0 человек

26–50%

0 человек

51–75%

0 человек

76–100%

0 человек

0% Пройти тест

Физика. Внутренняя энергия. Количество теплоты. 8 класс

Статистика теста

0–25%

6 человек

26–50%

17 человек

51–75%

14 человек

76–100%

16 человек

0% Пройти тест

Физика. Введение. Первоначальные сведения о строении вещества. 7 класс

Статистика теста

0–25%

6 человек

26–50%

60 человек

51–75%

92 человека

76–100%

94 человека

0% Пройти тест

Физика. Инерциальные системы отсчета. Первый закон Ньютона. 9 класс

Статистика теста

0–25%

1835 человек

26–50%

1753 человека

51–75%

1798 человек

76–100%

3583 человека

0% Пройти тест

все тесты

Что? Где? Когда? Эрудит онлайн: ответы на вопросы:

  • В каком случае виден тот или иной предмет?
  • Какие из указанных ниже групп японского населения относились к первому сословию?
  • Сухие дрова, масса которых 0,44 т, или нефть массой 100 кг выделит при полном сгорании большее количество теплоты?
  • В каком ряду все слова являются стилистически окрашенными?
  • У каких животных личинка проходит стадию куколки?
  • Каких частиц в атоме равное число?
  • Какое положение не относится к сильным сторонам локально-цивилизационного подхода к развитию общества?
  • В каких районах океана образуются льды?
  • Как называется отношение работы, совершаемой электрическим полем при перемещении положительного заряда, к значению заряда?
  • Как будет выглядеть в виде двойного неравенства: x больше или равно -5 и меньше 1?
  • Зависимость силы тока от каких физических величин устанавливает закон Ома?
  • В каком море больше воды — в Черном или Балтийском?
  • Что больше: пройденный вами путь или модуль перемещения?»> Утром вы выходите из дома, а вечером снова возвращаетесь. Что больше: пройденный вами путь или модуль перемещения?
  • С помощью каких уже известных вам измерительных приборов можно определить мощность электрического тока?
  • Наличие каких составных частей обязательно для работы теплового двигателя?

Вид магнитного поля вокруг провода

4 вариант

1. Вокруг каких зарядов образуется магнитное поле?

1) Любых электрических зарядов
2) Вокруг всех однонаправленно движущихся зарядов
3) Только вокруг упорядоченно перемещающихся электронов
4) Вокруг неподвижных электрических зарядов

2. О чем свидетельствует опыт Эрстеда?

1) О нагревании проводника электрическим током
2) Об отклонении магнитной стрелки, находящейся около проводника с током, в определенную сторону
3) О существовании вокруг проводника с током магнитного поля.

3. Правильно ли обозначено на рисунке направление линий магнитного поля проводника с током?

1) Да
2) Нет
3) Определить нельзя, так как неизвестно направление тока в проводнике

4. Чем катушка с током похожа на магнитную стрелку?

1) Наличием полюсов
2) Формой
3) Тем, что тоже имеет два полюса и при возможности свобод­но поворачиваться устанавливается в направлении «север — юг»

5. Каким образом можно изменить направление магнитного поля катушки на противоположное?

1) Изменив направление тока и перевернув ее полюсы
2) Изменив направление тока или поменяв местами ее по­люсы
3) Повысив напряжение или силу тока в катушке

6. Какой из названных ниже приборов применяется для регули­рования силы притяжения магнитом железных предметов?

1) Магнитная стрелка
2) Амперметр
3) Вольтметр
4) Реостат

7. Притягивается к магниту

1) резина
2) шерсть
3) сталь
4) шелк

8. Как называют явление существования необычно сильного магнитного поля в какой-либо местности Земли?

1) Магнитное поле Земли
2) Магнитная буря
3) Магнитное взаимодействие
4) Магнитная аномалия

9. Укажите, какая из картин магнитного поля, изображенных на рисунках, соответствует магнитному полю катушки с то­ком.

1) №1
2) №2
3) №3

10. Какой полюс магнита — северный или южный — расположен слева?

1) На обоих рисунках — северный
2) На обоих рисунках — южный
3) На рис. №1 — северный, на рис. №2 — южный
4) На рис. №1 — южный, на рис. №2 — северный

11. Что представляет собой электромагнит?

1) Навитый на каркас провод
2) Катушку с проволочной обмоткой и сердечником из железа :
3) Катушку с проволочной обмоткой и сердечником из намагниченного материала
4) Катушку с любым сердечником

12. Как изменится направление движения проводника с током под действием магнитного поля, если переключить полюсы источника тока и поменять местами полюсы магнитов?

1) Не изменится
2) Изменится на обратное
3) Проводник не придет в движение
4) Среди ответов нет правильного

13. При каком условии магнитное поле действует на проводник?

1) Если он заряжен
2) Если по нему течет ток
3) Если в нем большая сила тока

14. Почему электродвигатели не применяются так широко, как двигатели внутреннего сгорания, в автомобилях?

1) Потому что их КПД ниже
2) Из-за того, что на трассах везде есть бензоколонки, а не станции зарядки аккумуляторов
3) Потому что современные аккумуляторы не обеспечивают электродвигатели энергией длительное время

15. Какой из названных здесь двигателей обладает наибольшим КПД?

1) Реактивный двигатель
2) Газовая турбина
3) Двигатель внутреннего сгорания
4) Электродвигатель

Ответы на тест по физике Электромагнитные явления1 вариант
1-2
2-1
3-2
4-3
5-4
6-2
7-3
8-1
9-3
10-2
11-4
12-4
13-1
14-2
15-32 вариант
1-3
2-4
3-4
4-2
5-1
6-1
7-2
8-3
9-3
10-1
11-2
12-2
13-2
14-3
15-13 вариант
1-3
2-4
3-3
4-3
5-3
6-2
7-4
8-1
9-4
10-2
11-1
12-2
13-3
14-4
15-24 вариант
1-2
2-3
3-3
4-3
5-2
6-4
7-3
8-4
9-2
10-1
11-2
12-1
13-2
14-3
15-4

Что такое магнитное поле, его свойства

Многие видели и держали в руках магниты. Легко заметить ту силу, которая возникает между ними.

Каждый магнит обладает двумя полюсами: противоположные притягиваются, а одинаковые отталкиваются. Кроме того, магниты всегда окружены областью, где эта сила возникает. Магнитные поля как раз и описывают такую силу.

Таким образом, магнитное поле — это концепция, которую используют, чтобы описать то, как сила распределяется в пространстве вокруг магнита и в нем самом

Впервые на это явление обратил внимание французский ученый Перегрин, а затем исследовали Ампер и Фарадей

Явление магнетизма и магнитных полей — одна из составляющих электромагнитных сил, которые для природы базовые. Появляется магнитное поле там, где происходит движение зарядов. Когда большие заряды двигаются с высокими скоростями, то сила магнитного поля возрастает.

Магнитное поле вокруг магнита

Какова природа магнитного поля? Существуют способы, которые организовывают движение зарядов так, чтобы они такое поле порождали. Например:

  • Можно пустить ток по проводнику, присоединенному к батарее. Если силу тока увеличивать (то есть наращивать количество движущихся зарядов), то пропорционально усилится и магнитное поле. Его сила будет уменьшаться пропорционально расстоянию от проводника. Данное явление называют закон Ампера.
  • Можно использовать свойства электронов. Они имеют отрицательный заряд и совершают движение вокруг ядра атомов, что и есть основой принципа работы постоянного магнита. Не все материалы получится намагнитить. Для этого необходимы один или несколько так называемых непарных электронов (обычно электроны всегда образуют пары). Например, у атома железа есть четыре непарных электрона, поэтому из такого материала получится хороший магнит.

Каждый кусочек любого материала состоит из миллиардов атомов. Когда они ориентируются в пространстве произвольно, то их поле угасает, даже при наличии непарных электронов. Только в стабильных веществах можно получить постоянную ориентацию электронов, то есть постоянный магнит или ферромагнетик.

Некоторым материалам для этой цели необходим внешний источник магнитного поля. Оно способно сориентировать вращение электронов и задать им нужное направление, но стоит исчезнуть внешнему полю, и общая ориентация тоже пропадет. Такие материалы получили название парамагнетиков.

Хороший пример парамагнетика — металлическая дверца холодильников. Сама по себе она не магнит, но может притягивать приложенные к ней магниты. Это свойство многие используют, когда с помощью магнита крепят к дверце холодильника список покупок или записку.

Экспериментально подтвержденные свойства магнитного поля таковы:

  • оно материальное, то есть существует в объективной реальности, даже если о нем не знаем;
  • его порождают лишь движущиеся электрические заряды, то есть любое движущееся заряженное тело окружено таким полем. Магнитные поля создаются и магнитами, но и в этом случае причина появления кроется в движении электронов. Переменные электрические поля также создают их;
  • обнаруживают данные поля, действуя некоторой силой на движущиеся электрические заряды или проводники с током;
  • в пространстве его распространение происходит со скоростью, которая равна скорости света в условиях вакуума.

Таким образом, магнитное поле, определение которому дали выше, — это явление загадочное и невидимое, но в то же время вполне объяснимое.

Применение магнитного поля

1. Электромагниты:

Магнитные свойства электромагнита зависят:

1) От металла, из которого изготовлен сердечник.

2) От числа витков проволоки, намотанной на катушку.

3) От величины силы тока в катушке.

Электромагниты имеют широкое применение:

  • в телефонах
  • для переноски тяжелых грузов в цехах
  • в автоматических устройствах
  • для сортировки изделий по цвету, по форме, по качеству обработки изделий
  • для очистки зерна от мелких металлических изделий (в сепараторах)
  • в телеграфах

2. Электродвигатели.

Вращение катушки с током в магнитном поле используется в устройстве электрического двигателя. Первый электрический двигатель создан русским физиком, Б.С. Якоби.

Устройство электродвигателя:

1. Якорь (состоит из металлического цилиндра, в пазах которого укладываются обмотки, сделанные вдоль боковой поверхности железного цилиндра)

2. Электромагнит (усиливает магнитное поле)

3. Коллектор (состоит из медных пластин, к которым припаиваются обмотки, находящиеся в пазах якоря)

4. Щётки (при помощи которых подаётся ток на пластины коллектора)

Двигатели постоянного тока нашли широкое применение на транспорте: электровоз, трамваи, троллейбусы. В промышленности применяют двигатели, работающие на переменном токе. Их используют в насосах для выкачивания нефти из скважин, в работе различных станков. Практически во всех бытовых приборах применяют электрические двигатели. Электрические двигатели имеют большое преимущество. При работе они не выделяют газа, дым, пар. Их можно установить в удобном месте на станке, на тележке электровоза, в электробритвах, в прокатных станах, кораблях. Коэффициент полезного действия электрических двигателей высокий. Достигает 98%. Такого высокого коэффициента не имеет никакой другой двигатель.

<a target=»_blank» href=»https://text.ru/antiplagiat/60524a772d705″><img src=»https://text. ru/image/get/60524a772d705/1″ alt=»Text.ru — 100.00%» title=»Уникальность данного текста проверена через Text.ru» border=»0″ width=»80″ height=»31″></a>

Магнитное поле

Люди только и делают, что говорят про какие-то магнитные бури, привозят магнитики на холодильник, ходят в походы с компасом, который показывает, где север, а где юг. В основе всего этого лежит магнитное поле.

Магнитное поле — это особый вид материи, который существует вокруг магнитов или движущихся зарядов.

У нее есть несколько условий для существования:

  • магнитное поле существует независимо от наших знаний о нем;
  • порождается только движущимся электрическим зарядом;
  • обнаружить магнитное поле можно по действию на движущийся электрический заряд (или проводник с током) с некоторой силой;
  • магнитное поле распространяется в пространстве с конечной скоростью, равной скорости света в вакууме.

Магнитное поле создается только движущимся электрическим зарядом? А как же магниты?

Атом состоит из ядра и вращающихся вокруг него электронов. Электроны могут вращаться по разным орбитам. На каждой орбите может находиться по два электрона, которые вращаются в разных направлениях.

Но у некоторых веществ не все электроны парные, и несколько электронов крутятся в одном и том же направлении, такие вещества называются ферромагнетиками. А поскольку электрон — заряженная частица, вращающиеся вокруг атома в одну и ту же сторону электроны создают магнитное поле. Получается миниатюрный электромагнит.

Если атомы вещества расположены в произвольном порядке, поля этих крошечных магнитиков компенсируют друг друга. Но если эти магнитные поля направить в одну и ту же сторону, то они сложатся — и получится магнит.

У любого магнита есть два полюса — северный и южный.

Любое магнитное поле описывается магнитными линиями, которые выходят из северного поля и приходят в южный. Эти линии всегда замкнуты, даже если у них бесконечная длина. Вот так это выглядит:

Как запомнить, что выходят магнитные линии из северного полюса, а приходят в южный?

Все просто — на севере жить никто не хочет. Многие люди переезжают туда, где теплее, зимуют в теплых краях, в общем — стремятся на юг. Магнитные линии тоже.

Северный полюс обозначается латинской буквой N (от английского слова North). А южный — буквой S (от английского слова South).

Важный нюанс
Мы привыкли к тому, что на географическом севере находится северный магнитный полюс и на него указывает синяя стрелка компаса. Однако это не совсем так.

Из физики магнетизма нам известно, что силовые линии магнитного поля входят в южный полюс магнита, а выходят из северного. Если вы посмотрите на картину силовых линий магнитного поля Земли, то увидите, что они входят в Землю в районе северного географического полюса у канадских берегов Северного Ледовитого океана, а выходят в районе южного географического полюса в Антарктиде. Значит, с точки зрения физики у Земли на севере расположен южный магнитный полюс, а на юге — северный. Такие полюсы называются «истинными».

Однако, вопреки законам физики, люди договорились, что для простоты будут называть тот магнитный полюс, который находится на севере, северным, а тот магнитный полюс, что на юге, — южным. Такие магнитные полюсы Земли называются «мнимыми».

Закон электромагнитной индукции

Закон электромагнитной индукции (закон Фарадея) звучит так:

ЭДС индукции в замкнутом контуре равна и противоположна по знаку скорости изменения магнитного потока через поверхность, ограниченную контуром.

Математически его можно описать формулой:

Закон Фарадея

Ɛi — ЭДС индукции

ΔФ/Δt — скорость изменения магнитного потока [Вб/с]

Знак «–» в формуле позволяет учесть направление индукционного тока. Индукционный ток в замкнутом контуре всегда направлен так, чтобы магнитный поток поля, созданного этим током сквозь поверхность, ограниченную контуром, уменьшал бы те изменения поля, которые вызвали появление индукционного тока.

Если контур состоит из ​N витков (то есть он — катушка), то ЭДС индукции будет вычисляться следующим образом.

Закон Фарадея для контура из N витков

Ɛi — ЭДС индукции

ΔФ/Δt — скорость изменения магнитного потока [Вб/с]

N — количество витков

Сила индукционного тока в замкнутом проводящем контуре с сопротивлением ​R​:

Закон Ома для проводящего контура

Ɛi — ЭДС индукции

I — сила индукционного тока

R — сопротивление контура

Если проводник длиной l будет двигаться со скоростью ​v​ в постоянном однородном магнитном поле с индукцией ​B​ ЭДС электромагнитной индукции равна:

ЭДС индукции для движущегося проводника

Ɛi — ЭДС индукции

B — магнитная индукция

v — скорость проводника [м/с]

l — длина проводника

Возникновение ЭДС индукции в движущемся в магнитном поле проводнике объясняется действием силы Лоренца на свободные заряды в движущихся проводниках. Сила Лоренца играет в этом случае роль сторонней силы.

Движущийся в магнитном поле проводник, по которому протекает индукционный ток, испытывает магнитное торможение. Полная работа силы Лоренца равна нулю.

Количество теплоты в контуре выделяется либо за счет работы внешней силы, которая поддерживает скорость проводника неизменной, либо за счет уменьшения кинетической энергии проводника.

Изменение магнитного потока, пронизывающего замкнутый контур, может происходить по двум причинам:

  • вследствие перемещения контура или его частей в постоянном во времени магнитном поле. Это случай, когда проводники, а вместе с ними и свободные носители заряда, движутся в магнитном поле
  • вследствие изменения во времени магнитного поля при неподвижном контуре. В этом случае возникновение ЭДС индукции уже нельзя объяснить действием силы Лоренца. Явление электромагнитной индукции в неподвижных проводниках, возникающее при изменении окружающего магнитного поля, также описывается формулой Фарадея

Таким образом, явления индукции в движущихся и неподвижных проводниках протекают одинаково, но физическая причина возникновения индукционного тока оказывается в этих двух случаях различной:

  • в случае движущихся проводников ЭДС индукции обусловлена силой Лоренца
  • в случае неподвижных проводников ЭДС индукции является следствием действия на свободные заряды вихревого электрического поля, возникающего при изменении магнитного поля.

Магнитное поле проводника с током

Электрический ток, протекающий по проводнику с током, создает в окружающем его пространстве магнитное поле. Чем больше ток, проходящий по проводнику, тем сильнее возникающее вокруг него магнитное поле.

Магнитные силовые линии этого поля располагаются по концентрическим окружностям, в центре которых находится проводник с током.

Направление линий магнитного поля вокруг проводника с током всегда находится в строгом соответствии с направлением тока, проходящего по проводнику.

Направление магнитных силовых линий можно определить по правилу буравчика: если поступательное движение буравчика (1) совпадает с направлением тока (2) в проводнике, то вращение его рукоятки укажет направление силовых линий (4) магнитного поля вокруг проводника.

При изменении направления тока линии магнитного поля также изменяют свое направление.

По мере удаления от проводника магнитные силовые линии располагаются реже. Следовательно, индукция магнитного поля уменьшается.

Направление тока в проводнике принято изображать точкой, если ток идет к нам, и крестиком, если ток направлен от нас.

Для получения сильных магнитных полей при небольших токах обычно увеличивают число проводников с током и выполняют их в виде ряда витков; такое устройство называют катушкой.

В проводнике, согнутом в виде витка, магнитные поля, образованные всеми участками этого проводника, будут внутри витка иметь одинаковое направление. Поэтому интенсивность магнитного поля внутри витка будет больше, чем вокруг прямолинейного проводника. При объединении витков в катушку магнитные поля, созданные отдельными витками, складываются. При этом концентрация силовых линий внутри катушки возрастает, т. е. магнитное поле внутри нее усиливается.

Чем больше ток, проходящий через катушку, и чем больше в ней витков, тем сильнее создаваемое катушкой магнитное поле. Магнитное поле снаружи катушки также складывается из магнитных полей отдельных витков, однако магнитные силовые линии располагаются не так густо, вследствие чего интенсивность магнитного поля там не столь велика, как внутри катушки.

Магнитное поле катушки с током имеет такую же форму, как и поле прямолинейного постоянного магнита: силовые магнитные линии выходят из одного конца катушки и входят в другой ее конец. Поэтому катушка с током представляет собой искусственный электрический магнит. Обычно для усиления магнитного поля внутрь катушки вставляют стальной сердечник; такую катушку называют электромагнитом.

Направление линий магнитной индукции катушки с током находят по правилу правой руки:

если мысленно обхватить катушку с током ладонью правой руки так, чтобы четыре пальца указывали направление тока в ее витках, тогда большой палец укажет направление вектора магнитной индукции.

Для определения направления линий магнитного поля, создаваемого витком или катушкой, можно использовать также правило буравчика:

если вращать ручку буравчика по направлению тока в витке или катушке, то поступательное движение буравчика укажет направление вектора магнитной индукции.

Электромагниты нашли чрезвычайно широкое применение в технике. Полярность электромагнита (направление магнитного поля) можно определить и с помощью правила правой руки.

Но не всё так просто

Чтобы беспроводная зарядка работала эффективно, катушки зарядки и телефона должны быть как можно ближе друг к другу. Идеально, если их центры совпадут — тогда потери будут минимальные. Но мы часто видим, как телефоны просто кладут на любое место площадки, а зарядка всё равно идёт как нужно. 

Чтобы это было возможно, производители делают несколько катушек на площадке. Например, можно расположить несколько катушек рядом друг с другом, а сверху в промежутки положить ещё несколько. Тогда телефон почти всегда будет лежать над одной из катушек и будет заряжаться.

Чтобы зарядка понимала, какую катушку нужно использовать, она по очереди даёт ток на каждую и таким образом определяет, к какой катушке телефон лежит ближе всего.

Ещё зарядки отличаются по мощности — например, есть зарядки на 5, 10 и 20 ватт. Чтобы зарядка понимала, на какую мощность рассчитан ваш телефон и вообще подходил ли эта зарядка к телефону, она с помощью того же магнитного поля обменивается с телефоном данными. Если всё в порядке — заряд идёт, если нет — то всё отключается или зарядка идёт на минимальной мощности.

«Электромагнитная индукция»

Электромагнитная индукция — это явление, которое заключается в возникновении электрического тока в замкнутом проводнике в результате изменения магнитного поля, в котором он находится. Это явление открыл английский физик М. Фарадей в 1831 г. Суть его можно пояснить несколькими простыми опытами.

Описанный в опытах Фарадея принцип получения переменного тока используется в индукционных генераторах, вырабатывающих электрическую энергию на тепловых или гидроэлектростанциях. Сопротивление вращению ротора генератора, возникающее при взаимодействии индукционного тока с магнитным полем, преодолевается за счет работы паровой или гидротурбины, вращающей ротор. Такие генераторы преобразуют механическую энергию в энергию электрического тока.

Вихревые токи, или токи Фуко

Если массивный проводник поместить в переменное магнитное поле, то в этом проводнике благодаря явлению электромагнитной индукции возникают вихревые индукционные токи, называемые токами Фуко.

Вихревые токи возникают также при движении массивного проводника в постоянном, но неоднородном в пространстве магнитном поле. Токи Фуко имеют такое направление, что действующая на них в магнитном поле сила тормозит движение проводника. Маятник в виде сплошной металлической пластинки из немагнитного материала, совершающий колебания между полюсами электромагнита, резко останавливается при включении магнитного поля.

Во многих случаях нагревание, вызываемое токами Фуко, оказывается вредным, и с ним приходится бороться. Сердечники трансформаторов, роторы электродвигателей набирают из отдельных железных пластин, разделенных слоями изолятора, препятствующего развитию больших индукционных токов, а сами пластины изготовляют из сплавов, имеющих высокое удельное сопротивление.

Электромагнитное поле

Электрическое поле, созданное неподвижными зарядами, является статическим и действует на заряды. Постоянный ток вызывает появление постоянного во времени магнитного поля, действующего на движущиеся заряды и токи. Электрическое и магнитное поля существуют в этом случае независимо друг от друга.

Явление электромагнитной индукции демонстрирует взаимодействие этих полей, наблюдаемое в веществах, в которых есть свободные заряды, т. е. в проводниках. Переменное магнитное поле создает переменное электрическое поле, которое, действуя на свободные заряды, создает электрический ток. Этот ток, будучи переменным, в свою очередь порождает переменное магнитное поле, создающее электрическое поле в том же проводнике, и т. д.

Совокупность переменного электрического и переменного магнитного полей, порождающих друг друга, называется электромагнитным полем. Оно может существовать и в среде, где нет свободных зарядов, и распространяется в пространстве в виде электромагнитной волны.

Классическая электродинамика — одно из высших достижений человеческого разума. Она оказала огромное влияние на последующее развитие человеческой цивилизации, предсказав существование электромагнитных волн. Это привело в дальнейшем к созданию радио, телевидения, телекоммуникационных систем, спутниковых средств навигации, а также компьютеров, промышленных и бытовых роботов и прочих атрибутов современной жизни.

Краеугольным камнем теории Максвелла явилось утверждение, что источником магнитного поля может служить одно только переменное электрическое поле, подобно тому, как источником электрического поля, создающим в проводнике индукционный ток, служит переменное магнитное поле. Наличие проводника при этом не обязательно — электрическое поле возникает и в пустом пространстве. Линии переменного электрического поля, аналогично линиям магнитного поля, замкнуты. Электрическое и магнитное поля электромагнитной волны равноправны.

Электромагнитная индукция в схемах и таблицах

(Явление электромагнитной индукции, опыты Фарадея, правило Ленца, закон электромагнитной индукции, вихревое электрическое поле, самоиндукция, индуктивность, энергия магнитного поля тока)

Дополнительные материалы по теме:

Конспект урока по физике в 11 классе «Электромагнитная индукция».

Следующая тема: «».

Графическое изображение полей

Магниты действуют друг на друга и на железосодержащие предметы посредством магнитного поля. Поле не имеет цвета, запаха, его нельзя ощущать. Это особый вид материи, который проявляется по его действию на другое поле или на физические тела.

Условно изображают магнитное поле с помощью силовых линий, так же, как электрическое поле.

Эти линии замкнуты, то есть не имеют ни начала, ни конца. Направление, куда показывают северные полюсы магнитных стрелок, попавших в поле магнита, принято за направление силовых магнитных линий поля. Таковым оказывается направление от северного полюса к южному.

Хотя изображение силовых линий принято за условное, они все же проявляются в простом опыте с железными опилками. Если положить магнит на лист бумаги и посыпать мелкими опилками из железа, то можно увидеть, как они выстроятся вдоль определенных линий, как маленькие магнитные стрелки.

 

Частота линий вокруг магнита различна. Это подчеркивает более сильное действие магнитного поля около полюсов, где силовые линии плотнее.

Указатель юга и севера – компас.

Полюсы магнитные

«Указатель юга» — так называли древние китайцы свое изобретение. Это был прибор в форме ложки, изготовленный из природного магнита. Ложка могла вращаться вокруг вертикальной оси.

Древний китайский компас. 

Ручка ложки указывала южное направление. Она была северным полюсом ложки-магнита.

Развитие науки не остановилось, и современные компасы уже имеют другой вид:

Разные виды компасов. 

Магнитная стрелка, главный элемент компаса, — это постоянный магнит и имеет два полюса. Конец стрелки, указывающий на географический Север, называют северным (N), а противоположный – южным (S) полюсом. Отсюда и название полюсов различных магнитов.

Раскраска магнитов в красный и синий цвета условна, реже используются и другие цвета. Существенным является то, что полюсы магнитов существуют только парами. Если распилить, например, полосовой магнит, получатся два полосовых магнита, и у них будет снова по два полюса: северный и южный.

В школьных лабораторных работах используются маленькие магниты на подставке, которые насаживаются на тонкую иглу и могут свободно вращаться вокруг этой иглы. Такие устройства называются магнитными стрелками, как подобие стрелок компасов.

С помощью стрелок изучается взаимодействие полюсов магнитов. Если приблизить стрелки друг к другу, они начинают поворачиваться и установятся по следующему правилу:

Земной шар является огромным магнитом, у которого есть свои полюсы. Но нельзя путать магнитные полюсы Земли с географическими. Согласно правилу, синий (северный) конец стрелки должен поворачиваться к Южному полюсу земного шара, так как притягиваются разноименные полюсы. Да, действительно, это так. Южный магнитный полюс Земли находится вблизи Северного географического полюса, но не в той же точке, а чуть в стороне, на острове Принца Уэльского. Северный магнитный полюс находится в Антарктиде, где и Южный географический.

Месторасположение магнитных полюсов Земли не остается постоянным. Полюсы смещаются на расстояние нескольких десятков километров в год.

Очень широк список областей, где применяются магниты:

  • автомобилестроение;
  • приборостроение;
  • автоматика;
  • телемеханика;
  • тормозные системы;
  • компасы;
  • медицина;
  • радиотехника;
  • электротехника.

От изучения природных магнитных явлений человек давно шагнул к элетромагнитным явлениям, без чего невозможно развитие знаний об электричестве и электрическом токе.

1 вокруг движущихся электрических зарядов существует. Магнитное поле движущегося заряда

Вокруг любого проводника с током, т.е. движущихся электрических зарядов, существует магнитное поле. Ток следует рассматривать как источник магнитного поля! Вокруг неподвижных электрических зарядов существует только электрическое поле, а вокруг движущихся зарядов – и электрическое, и магнитное. ХАНС ЭРСТЕД ()

1. Магнитное поле возникает только около движущихся электрических зарядов. 2. Оно ослабевает по мере удаления от проводника с током (или движущегося заряда) и точных границ поля указать нельзя. 3. Действует на магнитные стрелки определённым образом 4. Обладает энергией и имеет свою внутреннюю структуру, которая отображается с помощью магнитных силовых линий. Магнитные линии магнитного поля тока представляют собою замкнутые линии, охватывающие проводник

Если контура с током последовательно соединить в одном месте пространства, то такое образование называется соленоидом. Магнитное поле сконцентрировано внутри соленоида, снаружи рассеяно, и магнитные силовые линии внутри соленоида параллельны между собой и поле внутри соленоида считается однородным, вне соленоида — неоднородным. Поместив внутрь соленоида стальной стержень, мы получим простейший электромагнит. При прочих равных условиях магнитное поле электромагнита гораздо сильнее магнитного поля соленоида.


Совпадают ли магнитные полюсы Земли с географическими полюсами? Менялось ли местоположение магнитных полюсов в истории планеты? Что является надёжным защитником жизни на Земле от космических лучей? В чём заключена причина появления магнитных бурь на нашей планете? С чем связаны магнитные аномалии? Почему магнитная стрелка имеет вполне определённое направление в каждом месте Земли? Куда она указывает?

Проверь себя!!! Вокруг движущихся зарядов эл.поле… Вокруг движущихся зарядов эл.поле… Электрический ток -… Электрический ток -… Постоянный эл.ток -… Постоянный эл. ток -… Два условия для возникновения эл.тока… Два условия для возникновения эл.тока… Сила тока -… Сила тока -… Амперметром измеряют… и включают его в цепь… Амперметром измеряют… и включают его в цепь… Вольтметром измеряют… и включают его… Вольтметром измеряют… и включают его… Вльт-амперная характеристика для металлов… Вльт-амперная характеристика для металлов… От чего зависит сопротивление проводника… От чего зависит сопротивление проводника… Закон Ома… Закон Ома… Через сечение проводника за 10с проходит заряд, равный 20Кл. Чему равна сила тока в цепи? Через сечение проводника за 10с проходит заряд, равный 20Кл. Чему равна сила тока в цепи? В сети напряжение 220В и сила тока 2А. Какое сопротивление может быть у прибора, который можно включить в эту сеть? В сети напряжение 220В и сила тока 2А. Какое сопротивление может быть у прибора, который можно включить в эту сеть?





Задача 2 Определите сопротивление участка цепи, при соединении в точках В и Д, если R1=R2=R3=R4=2 Ом Определите сопротивление участка цепи, при соединении в точках В и Д, если R1=R2=R3=R4=2 Ом Изменится ли сопротивление участка цепи при соединении в точках А и С? Изменится ли сопротивление участка цепи при соединении в точках А и С? Дано: R1=2 Ом R2=2 Ом R3=2 Ом R4=2 Ом Найти: Rоб-? Решение: R1,4=R1+R4, R1,4=2+2=4 (Ом) R2,3=R2+R3, R2,3=2+2=4 (Ом) 1/Rоб= 1/R1,4+ 1/R2,3, 1\Rоб=1/4+1/4=1/2 Rоб=2 (Ом) Ответ: Rоб=2 Ом.


Дано: R1=0,5 ОмR2=2 ОмR3=3,5 ОмR4=4 ОмRоб=1 Ом Дано: R1=0,5 ОмR2=2 ОмR3=3,5 ОмR4=4 ОмRоб=1 Ом Определить способ соединения. Определить способ соединения.Решение: R1,3=R1+R3, R1,3=0,5+3,5=4(Ом) R1,3,4=…; R1,3,4=2 (Ом) Rоб=1 (Ом) Значит R1,3-последовательно, R1,3 и R4-параллельно, R1,3,4 и R2- параллельно.


Рассмотрим как соединены 1,2,3 резисторы? Можем мы для них рассчитать R об? 1/R I =1/R 1 +1/R 2 +1/R 3 ; R I =1 Ом. А теперь посмотрите, как соединены эти три резистора с четвертым? Значит я могу заменить 1,2,3 резисторы на одно сопротивление R I =1 Ом, которое эквивалентно трем резисторам соединенным параллельно. Какой тогда будет схема соединения? Начертите ее. Как теперь найти общее сопротивление? R Об =R I +R 4 ; R Об =1 Ом +5 Ом=6 Ом Теперь осталось решить вопрос чему же равно общая сила тока при таком соединении? I об =I=I 4, следовательно Uоб=5 А*6 Ом=30 В Запишем ответ к задачи.


> >R 3,4 =1 Ом. R об — ? U АВ — ? 2. Перейдем к эквивалентной схеме 3. R 1, R 2 и R 3,4 соединены последовательно > R об = R 1 +R 2 + R 3,4 > R об =5 Ом 4. U АВ » title=»Дано: R 1 = R 2 = R 3 = R 4 = 2 Ом I = 6 А Решение: 1.R 3 и R 4 соединены параллельно, > > >R 3,4 =1 Ом. R об — ? U АВ — ? 2. Перейдем к эквивалентной схеме 3. R 1, R 2 и R 3,4 соединены последовательно > R об = R 1 +R 2 + R 3,4 > R об =5 Ом 4. U АВ «> 13 Дано: R 1 = R 2 = R 3 = R 4 = 2 Ом I = 6 А Решение: 1.R 3 и R 4 соединены параллельно, > > >R 3,4 =1 Ом. R об — ? U АВ — ? 2. Перейдем к эквивалентной схеме 3. R 1, R 2 и R 3,4 соединены последовательно > R об = R 1 +R 2 + R 3,4 > R об =5 Ом 4. U АВ =U 1 +U 2 +U 3,4, где, > или > U АВ =6 А ·5 Ом=30 В Ответ: U АВ = 30 В > >R 3,4 =1 Ом. R об — ? U АВ — ? 2. Перейдем к эквивалентной схеме 3. R 1, R 2 и R 3,4 соединены последовательно > R об = R 1 +R 2 + R 3,4 > R об =5 Ом 4. U АВ «> > >R 3,4 =1 Ом. R об — ? U АВ — ? 2. Перейдем к эквивалентной схеме 3. R 1, R 2 и R 3,4 соединены последовательно > R об = R 1 +R 2 + R 3,4 > R об =5 Ом 4. U АВ =U 1 +U 2 +U 3,4, где, > или > U АВ =6 А ·5 Ом=30 В Ответ: U АВ = 30 В»> > >R 3,4 =1 Ом. R об — ? U АВ — ? 2. Перейдем к эквивалентной схеме 3. R 1, R 2 и R 3,4 соединены последовательно > R об = R 1 +R 2 + R 3,4 > R об =5 Ом 4. U АВ » title=»Дано: R 1 = R 2 = R 3 = R 4 = 2 Ом I = 6 А Решение: 1.R 3 и R 4 соединены параллельно, > > >R 3,4 =1 Ом. R об — ? U АВ — ? 2. Перейдем к эквивалентной схеме 3. R 1, R 2 и R 3,4 соединены последовательно > R об = R 1 +R 2 + R 3,4 > R об =5 Ом 4. U АВ «> title=»Дано: R 1 = R 2 = R 3 = R 4 = 2 Ом I = 6 А Решение: 1.R 3 и R 4 соединены параллельно, > > >R 3,4 =1 Ом. R об — ? U АВ — ? 2. Перейдем к эквивалентной схеме 3. R 1, R 2 и R 3,4 соединены последовательно > R об = R 1 +R 2 + R 3,4 > R об =5 Ом 4. U АВ «>

По горизонтали: 1. Отрицательно заряженная частица, входящая в состав атома. 2. Нейтральная частица, входящая в состав атомного ядра. 3. Физическая величина, характеризующая противодействие, оказываемое проводником электрическому току. 4. Единица электрического заряда. 5. Прибор для измерения силы тока. 6. Физическая величина, равная отношению работы тока к переносимому заряду. По вертикали: 1. Процесс сообщения телу электрического заряда. 2. Положительно заряженная частица, входящая в состав атомного ядра. 3. Единица напряжения. 4. Единица сопротивления. 5. Атом, присоединивший или потерявший электрон. 6. Направленное движение заряженных частиц. 6. Направленное движение заряженных частиц.

Создает вокруг себя, является более сложным, чем то, что свойственно заряду, находящемуся в неподвижном состоянии. В эфире, где пространство не возмущено, заряды уравновешиваются. Поэтому он называется магнитно- и электрически нейтральным.

Рассмотрим более подробно поведение такого заряда отдельно, в сравнении с неподвижным, и подумаем о принципе Галилея, а вместе с тем и о теории Эйнштейна: насколько она состоятельна на самом деле?

Различие движущегося и неподвижного зарядов

Одиночный заряд, будучи неподвижным, создает электрическое поле, которое можно назвать результатом деформации эфира. А движущийся электрический заряд создает как электрическое, так и Он обнаруживается только другим зарядом, то есть магнитом. Получается, что покоящийся и движущийся заряды в эфире не эквивалентны друг другу. При равномерном и заряд не будет излучать и не будет терять энергию. Но так как часть ее тратится на создание магнитного поля, то энергии у этого заряда станет меньше.

Пример для облегчения понимания

Это легче представить на примере. Если взять два одинаковых неподвижных заряда и расположить их далеко друг от друга, чтобы поля не могли взаимодействовать, один из них оставят как есть, а другой будут перемещать. Для первоначально неподвижного заряда потребуется ускорение, которое будет создавать магнитное поле. Часть энергии этого поля уйдет на электромагнитное излучение, направленное в бесконечное пространство, которое уже не вернется в качестве самоиндукции при остановке. С помощью другой части зарядной энергии будет создаваться постоянное магнитное поле (при условии постоянной скорости заряда). Это энергия деформации эфира. При магнитное поле сохранится в постоянном виде. Если при этом сравнить два заряда, то у движущегося будет наблюдаться меньшее количество энергии. Всему виной движущегося заряда, на которое ему приходится тратить энергию.

Таким образом, становится понятным, что в обоих зарядах состояние и энергия сильно отличаются. Электрическое поле действует на неподвижные и на движущиеся заряды. Но на последний влияет и магнитное поле. Поэтому и энергия, и потенциал у него меньше.

Движущиеся заряды и принцип Галилея

Состояние обоих зарядов можно также отследить в подвижном и неподвижном физическом теле, которое не имеет движущихся заряженных частиц. И принцип Галилея здесь может быть объективно провозглашен: физическое и нейтральное к электричеству тело, которое двигается равномерно и прямолинейно, неотличимо от того, что находится в покое по отношению к Земле. Получается, что нейтральные к электричеству тела и заряженные проявляют себя по-разному в состоянии покоя и в движении. Принцип Галилея не может использоваться в эфире и не может применяться к подвижным и неподвижным заряженным телам.

Несостоятельность принципа для заряженных тел

Теорий и работ о тех полях, что создает движущийся электрический заряд, сегодня накопилось немало. К примеру, Хэвисайд показал, что электрический вектор, образованный зарядом, является радиальным повсюду. Силовые магнитные линии, которые образованы точечным зарядом при движении, являются кругами, а в их центрах находятся линии движения. Другой ученый, Серл, решил задачу о распределении заряда в сфере, пребывающей в движении. Было выяснено, что оно порождает поле, подобное тому, что и движущийся электрический заряд создает, несмотря на то что последний — не сфера, а сжатый сфероид, в котором полярная ось направлена в сторону движения. Позже Мортон показал, что в наэлектризованной сфере, пребывающей в движении, плотность на поверхности меняться не будет, однако силовые линии уже не будут ее покидать под углом в 90 градусов.

Энергия, окружающая сферу, становится больше при ее движении, чем в то время, когда сфера покоится. Это происходит потому, что кроме электрического поля, вокруг движущейся сферы также появляется магнитное поле, как и в случае с зарядом. Поэтому, чтобы выполнить работу, скорость для заряженной сферы потребуется большая, чем для той, что является нейтральной электрически. Вместе с зарядом возрастет и эффективная масса сферы. Авторы уверены, что это происходит из-за самоиндукции конвекционного тока, который движущийся электрический заряд создает с начала движения. Таким образом, принцип Галилея признается несостоятельным для заряженных электричеством тел.

Идеи Эйнштейна и эфир

Тогда становится понятным и то, почему Эйнштейн не выделял место эфиру в СТО. Ведь сам факт признания наличия эфира уже разрушает принцип, заключающийся в эквивалентности инерциальных и независимых систем отсчета. А он, в свою очередь, и является основой СТО.

На вопрос Магнитное поле образуется движущимся зарядом? заданный автором Двутавровый лучший ответ это Все именно так. Движение относительно. Поэтому магнитное поле будет наблюдаться в той системе, относительно которой движется заряд. Чтобы получилось магнитное поле, вовсе не обязательно движение двух разноименно заряженных частиц. Просто при протекании тока в проводниках заряды скомпенсированы и более слабые (по сравнению с электростатическими) магнитные эффекты выходят на первый план.
Расчеты по выводу уравнений магнитных полей из СТО и кулоновского поля можно найти в любом учебнике по электродинамике. Например в Фейнмановских лекциях по физике, т. 5 (Электричество и магнетизм) Гл. 13 (Магнитостатика) в §6 подробно рассмотрен как раз этот вопрос.
Учебник можно найти по ссылке http:// lib. homelinux. org/_djvu/P_Physics/PG_General courses/Feynman/Fejnman R., R.Lejton, M.Se»nds. Tom 5. E»lektrichestvo i Magnetizm (ru)(T)(291s).djvu
Много интересного есть и в 6-м томе (Электродинамика) .
http:// lib. homelinux. org/_djvu/P_Physics/PG_General courses/Feynman/Fejnman R., R.Lejton, M.Se»nds. Tom 6. E»lektrodinamika (ru)(T)(339s).djvu
(уберите только лишние пробелы в адресе сайта)
А излучение и магнитное поле от заряженной палочки, которой размахиваете, будет малым не из-за скорости, а из-за ничтожности заряда (и величины тока, создаваемого движением столь малого заряда — можете посчитать сами).

Ответ от Просачиваться [гуру]
Само понятие движения относительно. Поэтому да, в одной системе координат магнитное поле будет, в другой оно будет другим, в третьей его вовсе не будет. На самом деле магнитного поля вообще нет, просто эффекты специальной теории относительности для движущихся зарядов удобно описывать введением фиктивного поля, называемого магнитным и сильно упрощающего расчеты. До появления теории относительности магнитное поле считали самостоятельной сущностью и лишь потом установили, что приписываемые ему силы прекрасно рассчитываются и без него на основе теории относительности и закона Кулона. Но, конечно, теорию относительности куда сложнее применять практически, чем правило буравчика 😉 И так как электрическое и магнитное поле оказываются тесно связаны (хотя второе — наглядная интерпретация следствий изменений первого) , говорят о едином электромагнитном поле.
А насчет бегания по комнате с заряженной палочкой, тут и теории относительности не надо — конечно, образуется магнитное поле, излучаются волны и так далее, только очень слабые. Посчитать напряженность создаваемого поля — задача для школьника.

Ответ от Посовеститься [гуру]
Ну вот, опять курил в туалете вместо физики.. . Учебник слабо открыть? Там чётко написано «электромагнитное поле» и т. д. и т. л. Любят у нас лисапеты сочинять да вечные двигатели придумывать. На торсионных полях..

Ответ от VintHeXer [активный]
Вообще ИМХО по закону Ампера и ещё какой-то очень умной формулы, имеющей в записи синус угла, уже показывает, что нужно движение заряженной частицы в проводнике (опять же ИМХО) , тк сила тока будет при напряжении и сопротивлении.. . Напряжение вродь как есть (частица то заряжена) , а вот сопротивление в вакууме.. .
А вообще хрен знает.. . Особенно про движение зар-ой частицы в вакууме))

Ответ от Krab Вark [гуру]
Ну, подробный вывод надо искать в учебниках физики. Такой можно скачать, например, тут:)
«хоть и с Вашей помощью – но дети постепенно выведут магнитное притяжение или отталкивание токов в электронейтральных проводниках из закона Кулона и теории относительности. Для них это будет чудо, сотворенное собственными руками. Большего в средней школе не требуется. В университете им небрежно объяснят, как из закона Кулона для неподвижных зарядов и формул преобразований квадратичных дифференциальных форм в теории относительности следуют уравнения электромагнитных полей Максвелла. «
А вообще надо в таких вопросах ставить галочку в поле возможности делать комментарии.. .

Магнитное поле на Википедии
Посмотрите статью на википедии про Магнитное поле

Магнитное поле движущегося заряда может возникать вокруг проводника с током. Так как в нем движутся электроны, обладающие элементарным электрическим зарядом. Также его можно наблюдать и при движении других носителей зарядов. Например, ионов в газах или жидкостях. Это упорядоченное движение носителей зарядов, как известно, вызывает в окружающем пространстве возникновение магнитного поля. Таким образом, можно предположить, что магнитное поле независимо от природы тока его вызывающего возникает и вокруг одного заряда находящегося в движении.

Общее же поле в окружающей среде формируется из суммы полей создаваемых отдельными зарядами. Этот вывод можно сделать исходя из принципа суперпозиции. На основании различных опытов был получен закон, который определяет магнитную индукцию для точечного заряда. Это заряд свободно перемещается в среде с постоянной скоростью.

Формула 1 — закон электромагнитной индукции для движущегося точечного заряда

Где r радиус-вектор, идущий от заряда к точке наблюдения

Q заряд

V вектор скорости движения заряда

Формула 2 — модуль вектора индукции

Где альфа это угол между вектором скорости и радиус вектором

Эти формулы определяют магнитную индукцию для положительного заряда. Если ее необходимо рассчитать для отрицательного заряда то нужно подставить заряд со знаком минус. Скорость движения заряда определяется относительно точки наблюдения.

Чтобы обнаружить магнитное поле при перемещении заряда можно провести опыт. При этом заряд не обязательно должен двигаться под действием электрических сил. Первая часть опыта состоит в том, что по проводнику круговой формы проходит электрический ток. Следовательно, вокруг него образуется магнитное поле. Действие, которого можно наблюдать при отклонении магнитной стрелки находящейся рядом с витком.

Рисунок 1 — круговой виток с током воздействует на магнитную стрелку

На рисунке изображён виток с током, слева показана плоскость витка справа плоскость перпендикулярная ей.

Во второй части опыта мы возьмем сплошной металлический диск, закрепленный на оси от которой он изолирован. При этом диску сообщен электрический заряд, и он способен быстро вращаться вокруг своей оси. Над диском закреплена магнитная стрелка. Если раскрутить диск с зарядом, то можно обнаружить что стрелка вращается. Причем это движение стрелки будет таким же, как при движении тока по кольцу. Если при этом изменить заряд диска или направление вращения, то и стрелка будет отклоняться в другую сторону.

Тест с ответами: “Магнитное поле”

1. Как взаимодействуют два параллельных проводника, если электрический ток в них протекает в одном направлении:
а) проводники притягиваются +
б) сила взаимодействия равна нулю
в) проводники отталкиваются

2. О чем свидетельствует опыт Эрстеда:
а) об отклонении магнитной стрелки около проводника с током
б) о существовании вокруг проводника с током магнитного поля +
в) о влиянии проводника с током на магнитную стрелку

3. Какая физическая величина имеет единицу 1 тесла:
а) взаимная индукция
б) магнитный поток
в) магнитная индукция +

4. Магнитные линии имеют начало и конец:
а) нет +
б) да
в) время от времени

5. Как изменится период обращения заряженной частицы в однородном магнитном поле при уменьшении ее скорости в 2 раза? Изменением массы частицы пренебречь:
а) уменьшится в 2 раза
б) увеличится в 2 раза
в) не изменится +

6. Однородное магнитное поле – поле, в любой точке которого сила действия на заряд одинакова по модулю и одинакова по направлению:
а) да
б) нет +
в) периодически

7. Магнитный поток через замкнутый виток, помещенный в однородное магнитное поле, зависит:
а) от размера витка
б) от расстояния между вектором магнитной индукции и нормалью к контуру
в) от модуля магнитной индукции +

8. Магнитный поток через замкнутый виток, помещенный в однородное магнитное поле, зависит:
а) от угла между вектором магнитной индукции и нормалью к контуру +
б) от размера витка
в) от расстояния между вектором магнитной индукции и нормалью к контуру

9. Магнитный поток через замкнутый виток, помещенный в однородное магнитное поле, зависит:
а) от размера витка
б) от расстояния между вектором магнитной индукции и нормалью к контуру
в) от площади витка +

10. В пространстве, где находится электрон, создается электрическое и магнитное поля:
а) да
б) нет +
в) зависит от условий

11. В каком случае вокруг движущегося электрона возникает магнитное поле:
1. Электрон движется равномерно и прямолинейно
2. Электрон движется равномерно по окружности
3. Электрон движется равноускоренно прямолинейно
а) только 1
б) 2 и 3
в) все варианты верны +
г) нет верного ответа

12. Девочка качается на качелях, держа в руках постоянный магнит. Магнитное поле обнаружится независимо от того, качели неподвижны или качаются:
а) нет
б) да +
в) только когда качаются качели

13. Вокруг каких зарядов, неподвижных или движущихся, существует электрическое поле:
а) электрическое поле существует вокруг всех зарядов +
б) магнитное поле существует вокруг неподвижных
в) электрическое поле существует вокруг движущихся

14. Вокруг каких зарядов, неподвижных или движущихся, существует магнитное поле:
а) электрическое поле существует вокруг движущихся
б) магнитное поле существует вокруг неподвижных
в) магнитное поле существует вокруг движущихся +

15. Что служит источником магнитного поля:
а) электрический ток +
б) электрический заряд
в) проводник, который включается в цепь

16. Магнитная линия магнитного поля:
а) линия, по которой движутся железные опилки
б) линия, вдоль которой устанавливаются в магнитном поле оси магнитных стрелочек +
в) линия, которая показывает действие магнитного поля на магнитные стрелочки

17. Какова форма магнитных линий магнитного поля прямого проводника с током:
а) замкнутые кривые вокруг проводника
б) радиальные линии, отходящие от проводника как от центра
в) концентрические окружности, охватывающие проводник +

18. Какое направление принято за направление магнитной линии магнитного поля:
а) направление, которое указывает южный полюс магнитной стрелки
б) направление, которое указывает северный полюс магнитной стрелки +
в) направление, в котором устанавливается ось магнитной стрелки

19. Что нужно сделать, чтобы магнитная стрелка, расположенная на магнитной линии магнитного поля прямого проводника с током, повернулась на 180°:
а) отклонить проводник от вертикального положения
б) отключить проводник от источника тока
в) изменить направление электрического тока в проводнике на противоположное +

20. Магнитное поле создаётся электрическим током или заряженными частицами, так ли это:
а) да
б) нет +
в) периодически

21. Взаимодействие проводников с током объясняется явлением электромагнитной индукции, верно ли утверждение:
а) нет +
б) да
в) отчасти

22. За направление вектора магнитной индукции принято направление от … полюса к … полюсу внутри магнита
а) северного полюса к южному
б) южного полюса к северному +
в) не имеет значения

23. У поверхности Земли магнитная стрелка не всегда показывает направление таких линий планеты:
а) ровных
б) электрических
в) магнитных +

24. Вектор магнитной индукции всегда ориентирован … току:
а) параллельно
б) перпендикулярно +
в) он не ориентирован току никак

25. Однородное магнитное поле – это поле, в любой точке которого сила действия на заряд одинакова по модулю и одинакова по направлению, так ли это:
а) да
б) отчасти
в) нет +

26. Магнитное поле – вихревое, т. к. векторные линии поля всегда:
а) разомкнуты
б) замкнуты +
в) параллельны

27. В тех областях пространства, где магнитное поле более сильное, магнитные линии изображают дальше друг от друга, верно ли утверждение:
а) нет +
б) да
в) отчасти

28. Силовое поле, действующее на движущиеся электрические заряды и на тела, обладающие магнитным моментом, независимо от состояния их движения:
а) ионическое поле
б) электрическое поле
в) магнитное поле +

29. Основной силовой характеристикой магнитного поля является:
а) вектор магнитной индукции +
б) вектор электро индукции
в) вектор физической индукции

30. Магнитное поле можно назвать особым видом материи, посредством которой осуществляется взаимодействие между движущимися заряженными частицами или телами, обладающими:
а) электрическим моментом
б) магнитным моментом +
в) электрическими волнами

1. Как называются линии, вдоль которых в магнитном поле располагаются оси небольших магнитных стрелок


Подборка по базе: Краткосрочный план урока. Типы линии,нанесение размеров.docx, Как называются космонавты в других странах.docx, Стандарты вдоль ЖЦ.docx, 3. Расчет и оценка уровня загрязнения почв вдоль автодорог (2).d, 05194-1 Ковригин Н Исследование распределения магнитного поля вд, Пищевыми добавками называются натуральные и синтетические вещест

1. Как называются линии, вдоль которых в магнитном поле располагаются оси небольших магнитных стрелок:
а) электрические
б) магнитные +
в) фокальными

2. В 1820 году Эрстед, изучая взаимодействие проводника с током и магнитной стрелкой, обнаружил, что вокруг проводника:
а) существует электрическое и магнитное поле +
б) существует электрическое поле
в) существует магнитное поле

3. Расположение в магнитном поле прямого тока железных опилок:
а) беспорядочно
б) по прямым линиям вдоль проводников
в) по замкнутым кривым, охватывающим проводник с током +

 

4. Какой из проводников называется соленоидом:
а) проводник в виде спирали, чья длина больше диаметра +
б) проводник любой формы
в) проводник в виде спирали, чья длина меньше диаметра

5. Что такое электромагнит:
а) катушка с большим количеством витком
б) катушка с железным сердечником внутри +
в) катушка большого диаметра

6. Как поменяется сила магнитного поля, если сила тока в цепи уменьшится:
а) не изменится никак
б) увеличится
в) уменьшится +

7. Если ввести железный сердечник в катушку, то магнитное поле:
а) усилится +
б) не изменится никак
в) уменьшится

8. Что нужно сделать, чтобы изменить магнитные полюсы электромагнита:
а) изменить род вещества обмотки
б) изменить направление тока в цепи +
в) магнитные полюса нельзя менять

9. Что произойдёт, если к северному полюсу постоянного магнита поднести северный полюс магнита:
а) произойдёт притяжение магнитов
б) магниты не будут взаимодействовать
в) произойдёт отталкивание магнитов +

10. У какого из полюсов проявляется наиболее сильное магнитное действие у магнита:
а) возле северного полюса
б) возле обоих полюсов +
в) возле южного полюса

11. Как называется неподвижная часть электродвигателя постоянного тока:
а) ротор
б) якорь
в) индуктор +

12. Как называется подвижная часть электрического двигателя постоянного тока:
а) якорь +
б) ротор
в) статор

13. Благодаря чему электрические двигатели получили наибольшее практическое применение:
а) имеют небольшие габариты
б) не имеют продуктов выделения
в) практически бесшумные +

14. Южный магнитный полюс Земли находится:
а) на экваторе
б) вблизи Северного географического полюса +
в) на Южном полюсе

15. Выберите места на нашей планете, в которых направление магнитной стрелки постоянно отклонено от направления магнитной линии Земли:
а) Южный географический полюс Земли
б) Северный географический полюс Земли
в) магнитные аномалии +

16. Северное сияние образуется при взаимодействии Земли с:
а) мелкими метеоритами
б) солнечный ветром +
в) кристаллами льда в атмосфере

17. Назовите главную теорию причины существования магнитного поля Земли:
а) движение расплавленных металлов с магмой земли и потоках металлических веществ в ядре планеты +
б) присутствие Луны
в) вращение планеты вокруг своей оси

18. Значительное возрастание солнечной активности:
а) тёмное пятно
б) магнитная буря +
в) солнечный ветер

19. Существование магнитного поля вокруг проводника с током, назовите опыт:
а) Опыт Кулона
б) Опыт Ома
в) Опыт Эрстеда +

20. Сколько полюсов у каждой катушки:
а) два +
б) северный
в) южный

21. Физическая величина, характеризующая энергоэффективность электрического двигателя:
а) масса и размеры
б) КПД двигателя +
в) мощность

22. Какое поле существует вокруг проводника, включённого в электроцепь:
а) электромагнитное +
б) магнитное
в) электрическое

23. Источник электрического поля:
а) отрицательный электрический заряд
б) положительный электрический заряд
в) любой электрический заряд +

24. Выберите форму магнитных линий магнитного поля прямого проводника с током:
а) замкнутые кривые вокруг проводника
б) концентрические окружности, охватывающие проводник +
в) отходящие от проводника радиальные линии

25. Каким образом можно усилить магнитное поле катушки с током:
а) увеличить силу тока в ней +
б) увеличить радиус катушки
в) сделать ее более длинной

26. Какое из перечисленных веществ не притягивается к магниту:
а) сталь
б) стекло +
в) кобальт

27. Что называется магнитной бурей:
а) резкое кратковременное изменение магнитного поля Земли +
б) изменение магнитного поля Земли
в) неожиданное усиление магнитного поля планеты

28. Назовите признак, по которому электродвигатели превосходят тепловые двигатели:
а) мощность
б) экологичность +
в) масса

29. Электрическое поле существует вокруг каких зарядов:
а) упорядоченно движущихся зарядов
б) неподвижных электрических зарядов
в) любых электрических зарядов +

30. Магнитное поле можно обнаружить вокруг зарядов:
а) в случае упорядоченного движения любых зарядов +
б) когда движутся отрицательные заряды
в) когда движутся положительные заряды

1. Что называют электрическим током:
а) направленное движение электронов
б) направленное движение заряженных частиц +
в) направленное движение частиц

2. Что необходимо, чтобы в проводнике возник электрический ток:
а) создание в проводнике электрического поля +
б) действие на электроны сил, вызывающих их движение
в) наэлектризовать проводник

3. Назначение источника тока:
а) создавать электрические заряды в проводнике
б) освобождать электроны в проводнике от связи с атомами
в) поддерживать существование в проводнике электрического поля +

 

4. Во всех источниках тока происходит этот процесс:
а) разделение положительно и отрицательно заряженных частиц +
б) скопление электронов или ионов
в) создание потоков заряженных частиц

5. Полюсами источника тока называется место, где:
а) электрические заряды взаимодействуют
б) разделяются электрические заряды
в) накапливаются электрические заряды разного знака +

6. Какие полюса и их количество имеет источник тока?
а) 3; положительный, отрицательный и нейтральный
б) 2; положительный и отрицательный +
в) 2; отрицательный и нейтральный

7. Для разделения в источнике тока электрических зарядов необходима такая энергия:
а) химическая
б) механическая
в) внутренняя
г) все ответы верны +

8. В гальваническом элементе разделение заряженных частиц происходит за счёт этой энергии:
а) энергии химических реакций +
б) энергии света
в) внутренней

9. Что в гальваническом элементе служит положительным электродом, а что — отрицательным:
а) положительным — слой смолы, отрицательным — цинковый сосуд
б) положительным — угольный стержень, отрицательным — цинковый сосуд +
в) положительным — угольный стержень, отрицательным — клейстер

10. Аккумулятор дает электрический ток только после того, как:
а) его зарядили от другого источника тока +
б) его согрели в теплом помещении
в) наэлектризовали его электроды

11. За направление электрического тока условно принимают то направление, по которому движутся в проводнике:
а) электроны и отрицательные ионы
б) отрицательные ионы
в) положительные ионы +

12. При протекании электрического тока через растворы солей в растворах выделяются вещества. В этом проявляется:
а) тепловое действие тока
б) химическое действие тока +
в) магнитное действие тока

13. При силе тока 4 А с электрическим сопротивлением 2 Ом, чему равно напряжение на участке цепи:
а) 8 В +
б) 2 В
в) 16 В

14. Если напряжение увеличить в 4 раза, а сопротивление остается неизменным, как изменится сила тока на участке цепи:
а) уменьшится в 4 раза
б) не изменится
в) увеличится в 4 раза +

15. Резисторы с сопротивлением 2 Ом и 3 Ом соединены последовательно. Необходимо выбрать верное утверждение:
а) сила тока в первом резисторе меньше, чем во втором
б) общее сопротивление резисторов больше 3 Ом +
в) общее сопротивление резисторов равно 1,2 Ом

16. Проволоку разрезали пополам и сложили вдвое. Её сопротивление изменится?
а) не изменится
б) увеличится в 4 раза
в) уменьшится в 4 раза +

17. Что необходимо для того чтобы в проводнике возник электрический ток:
а) только создать в нем электрическое поле +
б) только наличие в нем свободных электрических зарядов
в) только иметь потребителя электрической энергии

18. Устройство плавкого предохранителя основано на действии электрического тока:
а) на химическом
б) на тепловом +
в) на магнитном

19. В электрическую цепь включены параллельно друг другу сопротивления 2 Ом и 3 Ом. По меньшему из них течет ток 6 А. По большему — течет ток:
а) 4 А +
б) 2 А
в) 3 А

20. Сила тока измеряется в:
а) Джоулях
б) Омах
в) Амперах +

21. Каким бывает электрический ток:
а) заряженным и незаряженным
б) кратковременным и долговременным +
в) быстрым и медленным

22. В обычных условиях газ:
а) не проводит электрический ток +
б) проводит электрический ток
в) может проводить или не проводить ток в зависимости от вида газа

23. Ток начинает существовать когда:
а) когда молекулы начинают двигаться хаотично
б) появляются свободные заряды +
в) когда молекулы газа начинают сталкиваться друг с другом

24. Какие разряды электрического тока существуют:
а) самостоятельные +
б) ограниченные
в) неограниченные

25. Какой разряд можно назвать самым слабым:
а) тлеющий
б) тихий +
в) искровой

26. Что называется рекомбинацией:
а) аналогичный процесс ионизации
б) происходит одновременно с ионизацией
в) обратный процесс ионизации +

27. В результате чего возникает электрический ток в газах:
а) ионизации +
б) понижения температуры
в) расщепления молекул

28. Какой из разрядов электрического тока применяется в бактерицидных лампах:
а) дуговой
б) искровой
в) тлеющий +

29. Как называются разряды, которые существуют, пока на них действует тело из вне:
а) тлеющие разряды
б) несамостоятельные разряды +
в) самостоятельные разряды

30. Что называется электрическим полем:
а) особый вид материи, проявляющий себя в действии на электрические заряды +
б) вид материи, имеющий электрические заряды
в) особый вид материи, проявляющий себя в действии на движущиеся электрические заряды

1. Что образуется вокруг неподвижных электрических зарядов в пространстве?
1) Магнитное поле.
2) Электрическое поле.+
3) Электромагнитное поле.
4) Вакуум.

2.Около чего наиболее сильно проявляется магнитное действие любого магнита?
1) Оси магнита.
2) Магнитной линии.
3) Магнитной аномалии.
4) Полюса магнита.+

3.Куда всегда указывает южный конец магнитной стрелки?
1) Северный географический полюс Земли.+
2) Южный магнитный полюс Земли.
3) Экватор.
4) На южный географический полюс Земли.

 

4.Почему металлические опилки около полюса магнита располагаются «ёжиком»?
1) Они испытывают кулоновское отталкивание.
2) «Торчащие» концы намагничены одинаково и поэтому отталкиваются .+
3) Не действует притяжение.
4) Сильнее проявляется магнитное взаимодействие.

5. Выберите верное утверждение о силовых линиях вихревого электрического и магнитного полей?
1) Силовые линии этих полей замкнуты+
2) Силовые линии этих полей разомкнуты
3) У магнитного поля силовые линии замкнуты, а у вихревого электрического разомкнуты
4) У вихревого электрического поля силовые линии замкнуты, а у магнитного разомкнуты

6.  Выберите верное утверждение о силовых линиях вихревого электрического и электростатического полей?
1) Силовые линии этих полей замкнуты
2) Силовые линии этих полей начинаются на положительных зарядах, а заканчиваются на отрицательных
3) У вихревого электрического поля силовые линии замкнуты; а у электростатического начинаются на положительных зарядах, а заканчиваются на отрицательных+
4) Силовые линии этих полей начинаются на отрицательных зарядах, а заканчиваются на положительных

7.Что такое силовые линии магнитного поля прямого тока?
1) Окружности.
2) Концентрические замкнутые линии, которые охватывают проводник с током.+
3) Кривые, которые располагаются около проводника.
4) Спиралевидные линии.

8. В каком случае становится возможным увеличение подъёмной силы электромагнита?
1) Если увеличить силу тока в его обмотке.+
2) Если заменить металлический сердечник на пластиковый.
3) Если уменьшить число витков в обмотке.
4) Если увеличить площадь сечения катушки.

9.Линии магнитного поля всегда…
1) Замкнуты, непрерывны, иногда пересекаются.
2) Незамкнуты, непрерывны, не пересекаются, выходят из северного и
заходят в южный полюс.+
3) Замкнуты, непрерывны, не пересекаются, выходят из северного и
заходят в южный полюс.
3) Замкнуты, непрерывны, не пересекаются, выходят из южного и
заходят в северный полюс.
4) Незамкнуты, непрерывны, не пересекаются, выходят из центра
магнита.

10. Соотнесите фамилию ученого и его наблюдение/открытие
А. Ампер
Б. Эрстед
В. Фарадей
Г. Ленц
1) взаимодействие магнитной стрелки и проводника с током;
2) определение индукционного тока в замкнутом проводящем кольце;
3) взаимодействие неподвижных электрических зарядов;
4) связь между изменением магнитного потока и возникновением индукционного тока в катушке;
Д. взаимодействие параллельных токов.
Ответ:  1-В, 2-А,3-Г,4-Б

11. Кто из данных ученых высказал предположение, что возле движущихся заряженных частиц возникает магнитное поле?
1) Гилберт
2) Эрстед
3) Ампер+
4) Фарадей

12. Кто был первым, кто высказал предположение о том, что если магнитное поле сквозь замкнутый проводящий контур меняется, то оно способно породить электрический ток?
1) Гилберт
2) Эрстед
3) Ампер
4) Фарадей+

13.Электромагнитная индукция – это явление порождения в пространстве…
1) электрического поля переменным магнитным;+
2) магнитного поля электрическим зарядом;
3) электрического тока в замкнутом проводнике;
4) магнитного поля движущимися зарядами.

14. Сколько составляет КПД мощных электродвигателей?
1) до 98%+
2) до 30 %
3) до 60%
4) до 40%

15. Превращения энергии , происходящие в процессе работы турбогенератора…
1) тепловая энергия топлива преобразуется в электрическую энергию;
2) тепловая энергия топлива преобразуется в магнитную энергию;
3) потенциальная энергия воды преобразуется в электрическую энергию тока;+
4) потенциальная энергия воды преобразуется в магнитную энергию.

1. Когда происходит электризация тел:
а) в результате химической реакции
б) при соприкосновении заряженного и незаряженного тела +
в) оба варианта правильные

2. Как изменится сила кулоновского взаимодействия двух точечных зарядов при уменьшении между ними расстояния в 3 раза и увеличении обоих зарядов в 3 раза:
а) увеличится в 81 раз +
б) уменьшится в 9 раз
в) увеличится в 9 раз

3. При изменении расстояния между двумя точечными электрическими зарядами сила взаимодействия уменьшилась в 16 раз. Как изменилось расстояние между зарядами:
а) уменьшилось в 2 раза
б) увеличилось в 4 раза
в) уменьшилось в 4 раза +

 

4. Какое из действий тока наблюдается, если намотать на гвоздь провод и присоединить проводники к аккумулятору, то гвоздь намагничивается:
а) магнитное действие +
б) химическое действие
в) тепловое действие

5. При Полярном сиянии наблюдается такое действие тока:
а) механическое
б) магнитное
в) световое +

6. Какие частицы расположены в узлах кристаллической решетки металлов и какой у них заряд:
а) электроны, имеющие отрицательный заряд
б) ионы, имеющие положительный заряд +
в) ионы, имеющие отрицательный заряд

7. В обычных условиях металлы электрически нейтральны. Это можно объяснить тем, что в них:
а) нет электрических зарядов
б) отрицательный заряд всех свободных электронов по абсолютному значению равен положительному заряду всех ионов +
в) нет верного ответа

8. Что условно принято за направление тока:
а) от «+» к «-» источника +
б) от «-» к «+» источника
в) нет верного ответа

9. Когда говорят о скорости распространения электрического тока в проводнике, то о какой скорости идет речь:
а) скорость движения отдельных электронов
б) скорость распространения электрического поля +
в) нет верного ответа

10. Силой тока называют физическую величину, которая определяется электрическим зарядом, проходящим через:
а) поперечное сечение проводника
б) единичное поперечное сечение проводника за одну секунду
в) поперечное сечение проводника за одну секунду +

11. Силу тока измеряет:
а) Амперметр +
б) Динамометр
в) Вольтметр

12. Необходимо указать единицу измерения силы тока:
а) Ньютон
б) Ампер +
в) Джоуль

13. В 25 мА столько Ампер:
а) 250 А
б) 2500 А
в) 0,025 А +

14. В 0,25 А столько миллиампер:
а) 250 мА +
б) 0,25 мА
в) 25 мА

15. В 0,25 мА столько микроампер:
а) 2,5 мкА
б) 0,25 мкА
в) 250 мкА +

16. Какие носители зарядов создают электрический ток в металлических проводниках:
а) отрицательные ионы
б) электроны +
в) положительные ионы

17. Ток в металлах можно обнаружить по такому действию:
а) механическому
б) магнитному
в) тепловому +

18. Как движутся в металле электроны:
а) от «-» к «+» +
б) от «+» к «-»
в) нет верного ответа

19. Величина, которая характеризует ток:
а) электрический заряд
б) удельное сопротивление
в) напряжение +

20. Электрический заряд измеряется в:
а) Ом.
б) Кл. +
в) А.

21. Каким прибором измеряется сопротивление в электрической цепи:
а) омметр +
б) вольтметр
в) электрический счетчик

22. В металлах в пространстве между атомами движутся:
а) положительные ионы
б) отрицательные ионы
в) свободные электроны +

23. Необходимо измерить силу тока в лампе и напряжение к ней. Как должен быть включен по отношению к лампе вольтметр:
а) параллельно +
б) последовательно
в) без разницы

24. Как поменяется яркость свечения электрической лампы при замене всех медных соединительных проводников на нихромовые:
а) увеличится
б) уменьшится +
в) не поменяется

25. Электрическая лампа рассчитана на напряжение 220 В и силу тока 0,45 А. Мощность тока в лампе равна:
а) 100 Вт
б) 4100 Вт
в) 99 Вт +

26. В паспорте электрической плитки имеется надпись:«0,55 кВт, 220 В». Сила тока при указанном напряжении равна:
а) 58,4 А
б) 2,5 А +
в) 5 А

27. Электрический паяльник рассчитан на напряжение 127 В и силу тока 0,5 А. Чему равна работа тока в паяльнике за 10 минут:
а) 38,1 кДж +
б) 2 кДж
в) 10,58 кДж

28. В каком из приведённых ниже случаев на практике используется параллельное соединение:
а) подключение амперметра
б) подключение вольтметра +
в) предохранители

29. Работа электрического тока измеряется в таких единицах:
а) Вт.
б) Дж.
в) кВт*час +

30. Прибор, которым в электрической цепи измеряется работа тока:
а) электрический счетчик +
б) вольтметр
в) ваттметр

Что такое электромагнитные поля?

Что такое электромагнитные поля?
    • Популярные темы
      • Загрязнение воздуха
      • Коронавирусная болезнь (COVID-19)
      • Гепатит
    • Данные и статистика »
      • Информационный бюллетень
      • Факты наглядно
      • Публикации
    • Найти страну »
    • А
    • Б
    • В
    • Г
    • Д
    • Е
    • Ё
    • Ж
    • З
    • И
    • Й
    • К
    • Л
    • М
    • Н
    • О
    • П
    • Р
    • С
    • Т
    • У
    • Ф
    • Х
    • Ц
    • Ч
    • Ш
    • Щ
    • Ъ
    • Ы
    • Ь
    • Э
    • Ю
    • Я
    • ВОЗ в странах »
      • Репортажи
    • Регионы »
      • Африка
      • Америка
      • Юго-Восточная Азия
      • Европа
      • Восточное Средиземноморье
      • Западная часть Тихого океана
    • Центр СМИ
      • Пресс-релизы
      • Заявления
      • Сообщения для медиа
      • Комментарии
      • Репортажи
      • Онлайновые вопросы и ответы
      • События
      • Фоторепортажи
      • Вопросы и ответы
    • Последние сведения
    • Чрезвычайные ситуации »
    • Новости »
      • Новости о вспышках болезней
    • Данные ВОЗ »
    • Приборные панели »
      • Приборная панель мониторинга COVID-19
    • Основные моменты »
    • Информация о ВОЗ »
      • Генеральный директор
      • Информация о ВОЗ
      • Деятельность ВОЗ
      • Где работает ВОЗ
    • Руководящие органы »
      • Всемирная ассамблея здравоохранения
      • Исполнительный комитет
    • Главная страница/
    • Центр СМИ/
    • Вопросы и ответы/
    • Вопросы и ответы/
    • Что такое электромагнитные поля?

    4 августа 2016 г. | Вопросы и ответы

    Определения и источники

    Электрические поля возникают за счет разницы напряжений: чем больше электрическое напряжение, тем более сильным будет возникающее поле. Магнитные поля возникают там, где проходит электрический ток: чем сильнее ток, тем сильнее магнитное поле. Электрическое поле есть даже при отсутствии электрического тока. Если имеется электрический ток, то сила магнитного поля будет меняться в зависимости от расхода электроэнергии, а сила электрического поля остается при этом постоянной.
    (Выдержка из брошюры «Электромагнитные поля», опубликованной Европейским региональным бюро ВОЗ в 1999 г. (серия справочных брошюр для местных органов власти по вопросам здоровья и окружающей среды; 32).

    Природные источники электромагнитных полей

    Электромагнитные поля (ЭМП) окружают нас повсюду, оставаясь при этом невидимыми человеческому глазу. Электрические поля образуются при возникновении в атмосфере электрических зарядов, вызванных грозой. Магнитное поле Земли заставляет иглу компаса всегда указывать направление «север–юг» и помогает птицам и рыбам ориентироваться в пространстве.

    Антропогенные (искусственные) источники электромагнитных полей

    Помимо ЭМП, возникающих за счет природных источников, в спектре электромагнитных полей есть и те, которые создаются антропогенными источниками: например, рентгеновские лучи, используемые для диагностирования переломов конечностей в результате спортивных травм. Электричество в каждой штепсельной розетке ведет к образованию сопутствующих ЭМП низкой частоты. Различные радиоволны более высокой частоты используются для передачи информации при помощи ТВ антенн, радиостанций или базовых станций мобильной связи.

     

    Что лежит в основе различий между электромагнитными полями?

    Одна из основных характеристик электромагнитного поля – это его частота или соответствующая длина волны. Поля различной частоты воздействуют на организм по-разному. Вы можете попытаться представить электромагнитные волны в виде череды регулярно повторяющихся волн огромной скорости, равной скорости света. Частота – это показатель, который просто указывает число колебаний или циклов в секунду, а термин «длина волны» используется для определения расстояния между следующими одна за другой волнами. Следовательно, длина и частота волны тесно взаимосвязаны: чем выше частота, тем короче длина волны.

    Проведение простого сравнения поможет лучше проиллюстрировать вышеизложенное: привяжите длинную веревку к дверной ручке, а свободный конец веревки держите в руке. Если вы будете медленно поднимать и опускать руку с веревкой, то образуется одна большая волна; если же движения будут более быстрыми, то это приведет к возникновению целой серии небольших волн. Длина веревки при этом остается постоянной, а значит, чем больше волн вы создадите (то есть, волн более высокой частоты), тем меньше будет расстояние между ними (то есть, длина волны будет короче).

     

    Что происходит с организмом человека под воздействием электромагнитных полей?

    Воздействие электромагнитных полей – это не новое явление. Однако, в течение XX века воздействие антропогенных электромагнитных полей в окружающей нас среде неуклонно возрастало по мере того, как увеличивающийся спрос на электроэнергию, непрерывно развивающиеся технологии и изменяющиеся формы социального поведения приводили к созданию все большего числа искусственных источников ЭМП. На каждого из нас воздействует целый комплекс слабых электрических и магнитных полей, как дома, так и на работе, в результате производства и передачи электроэнергии, использования бытовой техники и промышленного оборудования, средств телекоммуникации и радио- и телевещания.

    Очень слабые электрические токи образуются в теле человека в результате химических реакций, происходящих в ходе нормального функционирования организма, даже при отсутствии внешних электрических полей. Например, нервы передают сигналы при помощи электрических импульсов. Большинство биохимических реакций (от биохимии пищеварения, до деятельности мозга) сопровождается перераспределением заряженных частиц. Даже сердце является электрически активным: ваш доктор может проследить это при помощи электрокардиограммы.

    Электрические поля низкой частоты воздействуют на организм человека точно так же, как на любой другой материал, состоящий из заряженных частиц. Когда электрические поля воздействуют на электропроводные материалы, они влияют на распределение электрических зарядов на поверхности таких материалов. Электрические поля являются причиной того, что электрический ток проходит через тело человека и уходит в землю.

    Низкочастотные магнитные поля индуцируют циркулирующие токи в организме человека. Сила этих токов зависит от интенсивности внешнего магнитного поля. Если токи достаточно сильные, они могут оказывать возбуждающее действие на нервы и мускулатуру, а также влиять на другие биологические процессы.

    Как электрические, так и магнитные поля могут индуцировать напряжение и токи в организме человека, но даже если человек находится непосредственно под высоковольтной линией электропередач (ЛЭП), индуцированные токи очень слабы в сравнении с пороговыми значениями для возникновения состояния шока или других последствий, обусловленных электричеством.

    Нагревание является основным биологическим эффектом от радиочастотных электромагнитных полей. Этот эффект использован в микроволновых печах для подогрева пищи. Уровни радиочастотных полей, воздействию которых обычно подвергаются люди, гораздо ниже уровней, способных вызвать значительное нагревание внутренних тканей организма.

    Ученые исследуют вероятность того, что при продолжительном воздействии поля ниже порогового уровня могут вызвать эффекты нагревания внутренних тканей организма. На сегодняшний день нет подтвержденных данных о неблагоприятных последствиях для здоровья от продолжительного слабого воздействия радиочастотных полей или полей промышленной частоты. Тем не менее, ученые продолжают активно заниматься научными исследованиями в этой области.

    Биологические эффекты или неблагоприятные последствия для здоровья? Что угрожает здоровью?

    Биологические эффекты – это поддающиеся измерению ответные реакции организма на раздражители или изменения в окружающей среде. Эти изменения необязательно вредны для вашего здоровья. Например, когда вы слушаете музыку, читаете книгу, едите яблоко или играете в теннис, возникает целый ряд биологических эффектов от этих процессов. Однако ни от одного из этих видов деятельности мы не ждем неблагоприятных последствий для здоровья.

    Организм обладает тонкими механизмами для того, чтобы подстроиться к множеству самых разных воздействий, которые мы испытываем в условиях окружающей среды. Постоянные изменения являются непременной частью нашей жизни. Однако нет сомнений в том, что организм не обладает адекватными компенсационными механизмами в отношении всех биологических эффектов. Изменения необратимого характера, создающие продолжительный по времени стресс для организма, могут представлять угрозу для здоровья.

    Неблагоприятное воздействие на здоровье вызывает поддающиеся обнаружению нарушения здоровья у человека, подвергшегося такому воздействию, или у его/ее детей; с другой стороны, биологические эффекты необязательно вызывают неблагоприятные последствия для здоровья.

    Неоспоримым является тот факт, что электромагнитные поля выше определенного уровня могут вызывать биологические эффекты. Эксперименты, проведенные на здоровых волонтерах, указывают на то, что кратковременное воздействие полей тех уровней, которые присутствуют в окружающей среде или в нашем доме, не вызывает явных пагубных последствий. В отношении воздействия ЭМП более высокого уровня, способных причинить вред здоровью, существуют строгие ограничения, сформулированные в национальных и международных руководствах. В настоящее время основные споры ведутся вокруг того, может ли продолжительное воздействие полей низких уровней вызвать биологические ответные реакции организма и повлиять на самочувствие людей.

    Широко распространенная обеспокоенность в отношении здоровья

    Взгляд на новостные заголовки последних лет позволяет нам в известной степени уяснить, какие вопросы вызывают общественную обеспокоенность. Последние десять лет в центре внимания, с точки зрения опасностей для здоровья, оказались многочисленные источники электромагнитных полей, в том числе, линии электропередач, микроволновые печи, компьютерные мониторы и экраны телевизоров, устройства безопасности, радары, а с недавних пор – мобильные телефоны и их базовые станции.

    Международный проект по ЭМП

    В ответ на растущую общественную обеспокоенность в связи с возможными неблагоприятными последствиями для здоровья человека от воздействия все увеличивающегося количества разнообразных источников ЭМП, в 1996 г. Всемирная организация здравоохранения (ВОЗ) приступила к реализации крупного мультидисциплинарного проекта по изучению ЭМП. Международный проект по ЭМП позволяет обобщить все современные научные знания и свести воедино ресурсы ведущих международных и национальных организаций и научных учреждений.

    Выводы научных исследований

    За последние 30 лет опубликовано около 25 тысяч статей по проблемам биологических эффектов и медицинского применения неионизирующего излучения. Несмотря на то, что некоторые люди считают, что следует и дальше наращивать научные исследования в этой области, на сегодняшний день научные знания в ней гораздо шире, чем знания в отношении большинства химических веществ. На основе недавно проведенного углубленного обзора научной литературы, ВОЗ пришла к выводу о том, что имеющиеся фактические данные не указывают на существование неблагоприятных последствий для здоровья от воздействия электромагнитных полей низких уровней. Однако в знаниях о биологических эффектах имеются определенные пробелы, что вызывает необходимость проведения дальнейших научных исследований.

    Последствия для общего состояния здоровья

    Некоторые представители общественности объясняют целый ряд симптомов «размытого» характера тем, что в домашних условиях мы подвергаемся слабому воздействию электромагнитных полей. К числу отмечаемых симптомов относятся следующие: головная боль, чувство беспокойства, суицидальные настроения и депрессия, тошнота, чувство усталости и потеря либидо. На сегодняшний день нет научно обоснованных данных, подтверждающих наличие связи между этими симптомами и воздействием электромагнитных полей. По крайней мере, некоторые из перечисленных проблем со здоровьем могут возникать из-за шума или других факторов окружающей среды, или из-за беспокойства, возникающего в связи с использованием новых технологий.

    Последствия для исхода беременности

    ВОЗ и другие организации провели оценку множества разнообразных источников электромагнитных полей и их воздействия в той среде, где мы живем и работаем, в том числе: компьютерных мониторов, гидростатических матрацев, одеял с электро-обогревом, радиочастотных сварочных аппаратов, оборудования для диатермии и радаров. В целом, совокупность фактических данных позволяет сделать вывод о том, что воздействие полей обычного для окружающей среды уровня не увеличивает риск какого-либо неблагоприятного исхода беременности, например спонтанного выкидыша, врожденных пороков развития, низкой массы тела при рождении или врожденных заболеваний. Время от времени поступают сообщения о взаимосвязи возникающих проблем со здоровьем и предположительным воздействием ЭМП. Например, имеются сообщения о случаях рождения недоношенных детей или детей с низкой массой тела при рождении в семьях людей, работающих в электронной промышленности. Однако научное сообщество не считает, что подобные случаи непременно связаны с воздействием полей (в отличие от связи, например, с таким фактором риска как воздействие растворителей).

    Катаракты

    Обычное раздражение глаз и катаракты иногда отмечаются у рабочих, испытывающих воздействие радиочастотного или микроволнового излучения высокого уровня. Однако исследования, проведенные на животных, не подтверждают предположение о том, что такие формы повреждения глаз могут быть вызваны полями тех уровней, которые не являются опасными с точки зрения возможного нагревания тканей организма. Нет фактических данных, подтверждающих, что подобные последствия имеют место при воздействии полей тех уровней, с которыми сталкивается обычное население.

    Электромагнитные поля и раковые заболевания

    Несмотря на многочисленные исследования данного вопроса, доказательства каких-либо возможных последствий такого рода от ЭМП остаются крайне противоречивыми. Однако, совершенно очевидно, что даже если ЭМП каким-то образом влияют на раковые заболевания, увеличение риска заболевания под воздействием ЭМП будет очень незначительным. Хотя имеющиеся на сегодняшний день результаты исследований отличаются крайней непоследовательностью, среди детей и взрослых не выявлено значительного увеличения риска заболевания любыми видами рака в результате воздействия ЭМП.

    Ряд эпидемиологических исследований позволяет предположить, что есть незначительное увеличение риска заболевания детей лейкемией под воздействием низкочастотных магнитных полей в домашних условиях. Однако ученые не делают общего вывода о том, что такие результаты указывают на наличие причинно-следственной связи между воздействием полей и заболеванием (напротив, можно говорить об искажениях, допущенных в исследованиях, или об эффектах, не связанных с воздействием полей). Частично, такое мнение явилось результатом того, что исследования на животных и лабораторные исследования не подтверждают наличия каких-либо воспроизводимых эффектов, согласующихся с гипотезой о том, что поля вызывают раковые заболевания или способствуют их возникновению. Сейчас в ряде стран проводятся широкомасштабные исследования, которые, возможно, помогут ответить на связанные с данной проблемой вопросы.

    Гиперчувствительность к электромагнитным полям и депрессия

    Некоторые люди сообщают о «гиперчувствительности» к электрическим или магнитным полям. Они задаются вопросом, не может ли чувство боли, головная боль, депрессия, сонливость, нарушения сна и даже судороги и эпилептические припадки объясняться воздействием электромагнитного поля.

    Научных данных, подтверждающих идею о гиперчувствительности к ЭМП, мало. В ходе недавних исследований, проведенных в скандинавских странах, был сделан вывод о том, что люди не демонстрируют стойких реакций на воздействие электромагнитных полей, если оно имеет место в должным образом контролируемых условиях. Не существует и какого-либо признанного биологического механизма для объяснения гиперчувствительности. Проведение научных исследований в данной области затруднено, поскольку здесь могут быть задействованы и другие субъективные ответные реакции организма помимо прямых эффектов от полей как таковых. Исследования в этом направлении продолжаются.

    Какова основная направленность текущих и будущих исследований?

    Сейчас значительные усилия сосредоточены на исследовании ЭМП в связи с раковыми заболеваниями. Продолжается изучение, хотя и в меньших масштабах, чем в конце 90-х годов, возможных канцерогенных эффектов от полей промышленной частоты.

    Долговременные неблагоприятные последствия для здоровья от мобильных телефонов – это еще одна область, в которой в настоящее время проводится много исследований. Очевидные неблагоприятные последствия от воздействия радиочастотных полей низких уровней не обнаружены. Однако, учитывая общественную обеспокоенность в отношении безопасности сотовых телефонов, дальнейшие исследования направлены на выяснение того, не могут ли иметь место менее очевидные последствия при очень низких уровнях воздействия.

    Основные положения

    • Широкий спектр факторов окружающей среды вызывает биологические эффекты. «Биологические эффекты» и «угрозы для здоровья» – это не одно и то же. Для выявления и оценки угроз для здоровья требуется проведение специального исследования.
    • На низких частотах внешние электрические и магнитные поля индуцируют слабые циркулирующие токи внутри организма человека. Практически во всех обычных условиях уровни индуцированных токов в организме слишком малы, чтобы вызвать явные последствия.
    • Основной эффект от радиочастотных ЭМП состоит в нагревании внутренних тканей организма.
    • Нет сомнений в том, что кратковременное воздействие очень мощных ЭМП может причинить вред здоровью. Сегодня общественную обеспокоенность в основном вызывают долговременные неблагоприятные последствия для здоровья от воздействия ЭМП более низкого уровня, чем тот, который обусловливает острые биологические реакции.
    • Международный проект по ЭМП был инициирован ВОЗ для того, чтобы получить научно-обоснованные и объективные ответы на вопросы, вызывающие общественную обеспокоенность, в отношении возможных угроз для здоровья от электромагнитных полей низких уровней.
    • Несмотря на широкомасштабные исследования, на сегодняшний день нет фактических данных, которые позволили бы сделать вывод о том, что воздействие ЭМП низких уровней вредит здоровью человека.
    • Международные исследования сосредоточены на изучении возможных связей между раковыми заболеваниями и ЭМП промышленного и радиочастотного диапазона.

     

    Результаты научных исследований

    Если электромагнитные поля (ЭМП) представляют угрозу для здоровья, последствия ощутят все индустриально-развитые страны. Общественность требует конкретных ответов на все более злободневный вопрос: могут ли ЭМП, с которыми мы сталкиваемся в повседневной жизни, вызывать неблагоприятные последствия для здоровья?

    Средства массовой информации нередко предлагают нам окончательные, с их точки зрения, ответы. Однако к таким сообщениям следует подходить с осторожностью, учитывая, что просвещение населения не является первоочередной задачей СМИ. Журналист может выбрать тему и написать статью, руководствуясь целым рядом причин далеко не технического характера: журналисты конкурируют между собой за время и место публикации, а журналы и газеты бьются за тираж.

    Оригинальные сенсационные заголовки, которые могут привлечь внимание максимального числа людей, помогают журналистам в достижении их целей, а плохая новость – это не только всегда большая новость, но нередко та единственная, о которой мы узнаем. Большое число исследований, которые наводят на мысль о безопасности ЭМП, в лучшем случае лишь слабо освещаются в СМИ. Наука не может дать гарантии полной безопасности, но в целом, продолжение научных исследований не может не обнадеживать.

    Необходимы различные виды исследований

    Совокупность исследований в различных областях крайне важна для оценки потенциального неблагоприятного воздействия электромагнитных полей на здоровье В различных видах исследований рассматриваются разные аспекты данной проблемы.

    Лабораторные исследования клеточного материала направлены на выявление основополагающих механизмов взаимосвязи между воздействием ЭМП и биологическими эффектами. Они проводятся для выявления механизмов, исходя из изменений на молекулярном и клеточном уровне, которые вызваны ЭМП. Такие изменения могут дать ключ к разгадке того, как физическая сила преобразуется в биологические процессы внутри организма человека. В рамках этих исследований отдельные клетки или ткани изымаются из привычной для них среды обитания, что может блокировать активность потенциальных компенсаторных механизмов.

    Исследования иного рода – с использованием животных – более близки к реальным жизненным ситуациям. В результате ученые получают фактические данные, имеющие более прямое отношение к определению безопасных уровней воздействия для человека. В таких исследованиях нередко изучаются несколько различных по уровню полей с тем, чтобы проследить зависимость «доза-эффект».

    Эпидемиологические исследования или исследования здоровья человека – это еще один непосредственный источник информации о долговременных последствиях воздействия ЭМП. Такие исследования направлены на изучение причин и распределения заболеваний в реальных жизненных ситуациях среди местных сообществ и профессиональных групп. Ученые пытаются определить, существует ли статистическая корреляция между воздействием ЭМП и заболеваемостью определенной болезнью или неблагоприятными последствиями для здоровья. Однако стоимость эпидемиологических исследований высока. Но что еще более важно, так это то, что они предусматривают проведение оценки очень сложных по составу групп населения, и обеспечить достаточно хороший контроль, необходимый для выявления малейших эффектов, в рамках таких исследований весьма непросто.

    Вот почему ученые проводят оценку всех релевантных фактических данных, когда принимают решение относительно потенциальных угроз для здоровья, включая данные эпидемиологических исследований, исследований на животных и исследований клеточного материала.

    Интерпретация результатов эпидемиологических исследований

    Эпидемиологические исследования сами по себе обычно не могут точно установить взаимосвязь между причинами и эффектами, прежде всего потому, что они определяют только статистическую корреляцию между воздействием и заболеванием, которое может быть или не быть результатом воздействия.

    Представим себе некое гипотетическое исследование, направленное на установление связи между воздействием ЭМП на рабочих-электриков компании «Х-Электрисити» и повышенным риском заболевания раком. Даже при выявленной статистической корреляции, она может объясняться неполными данными в отношении других факторов на рабочем месте. Например, рабочие-электрики могли испытать воздействие химических растворителей, способных вызывать раковые заболевания. Более того, наблюдаемая статистическая корреляция может быть результатом чисто статистических эффектов, или несовершенства схемы исследования.

    Вот почему нахождение взаимосвязи между каким-то фактором и определенным заболеванием не всегда означает, что именно этот фактор вызвал заболевание. Для установления причинно-следственной связи исследователь должен учитывать многие факторы. Аргументы в пользу наличия такой связи становятся более убедительными, если наблюдается постоянная и сильная корреляция между воздействием и эффектом, четкая зависимость «доза-эффект», убедительное объяснение биологического характера, если результаты подкреплены релевантными исследованиями на животных, а самое главное, если различные исследования согласуются друг с другом.

    Эти условия, как правило, не соблюдаются для исследований в области ЭМП и раковых заболеваний. Это одна из главных причин того, что ученые обычно не склонны делать вывод о наличии последствий для здоровья от слабых ЭМП.

    Почему сложно полностью исключить возможность присутствия весьма незначительных рисков?

    «По-видимому, отсутствие фактических данных о пагубных эффектах не может удовлетворить современное общество. Напротив, фактические данные об отсутствии таких эффектов все в большей степени востребованы». (Barnabas Kunsch, Австрийский научно-исследовательский центр Зайберсдорф)

    «Отсутствуют убедительные фактические данные о неблагоприятных последствиях от ЭМП для здоровья» или «Причинно-следственная связь между ЭМП и раковыми заболеваниями не подтверждена» – вот типичные формулировки тех выводов, к которым пришли экспертные комитеты, изучавшие данную проблему. Все это звучит так, как будто научное сообщество избегает ответа на интересующий всех вопрос. Зачем же тогда продолжать научные исследования, если ученые уже продемонстрировали, что никаких последствий нет?

    Ответ прост: научные исследования здоровья человека очень хорошо зарекомендовали себя с точки зрения выявления значительных эффектов, например, взаимосвязи между курением и раковыми заболеваниями. К сожалению, ученым сложнее отличить слабые эффекты от отсутствия эффектов как таковых. Если бы ЭМП тех уровней, которые типичны для окружающей среды, были сильными канцерогенными факторами, то к настоящему моменту было бы совсем просто продемонстрировать такую взаимосвязь.

    Напротив, если ЭМП низких уровней являются слабыми канцерогенами, или даже если они являются сильными канцерогенами для небольшой группы людей, живущих в крупном сообществе, такую взаимосвязь гораздо сложнее продемонстрировать. Более того, даже если крупное научное исследование укажет на отсутствие такой корреляции, мы никогда не сможем быть совершенно уверены в том, что такой взаимосвязи действительно не существует.

    Отсутствие эффекта может означать, что действительно эффектов нет. Но с тем же успехом это может свидетельствовать о том, что эффект просто не выявляется при помощи нашего метода оценки. Поэтому отрицательные результаты обычно менее убедительны, чем веские положительные результаты.

    Наиболее сложная ситуация возникла, к сожалению, в области эпидемиологических исследований в отношении ЭМП, и состоит она в том, что имеется целый ряд исследований, давших неубедительные положительные результаты, которые, при этом, носят взаимно противоречивый характер. В такой ситуации, сами ученые, вероятно, расходятся во мнении относительно важности полученных данных. Тем не менее, в силу изложенных выше причин, большинство ученых и медицинских работников согласны с тем, что даже если существуют какие-либо последствия для здоровья от ЭМП низкого уровня, они, скорее всего, крайне незначительны в сравнении с другими рисками для здоровья, с которыми люди сталкиваются в повседневной жизни.

    Что нас ждет в будущем?

    Основная цель международного проекта ВОЗ по ЭМП состоит в том, чтобы инициировать и скоординировать проведение научных исследований во всем мире для осуществления обоснованных ответных действий в связи с проблемами, вызывающими общественную обеспокоенность. Это позволит обобщить результаты исследований клеточного материала, исследований на животных, а также исследований здоровья человека для обеспечения наиболее всесторонней оценки рисков для здоровья. Целостная оценка результатов ряда релевантных и заслуживающих доверия исследований даст наиболее достоверный ответ относительно неблагоприятных последствий для здоровья (если таковые существуют) от продолжительного воздействия слабых электромагнитных полей.

    Один из способов иллюстрации необходимости получения фактических данных в результате проведения разнообразных экспериментов – это провести сравнение с разгадыванием кроссворда. Например, мы имеем девять вопросов, на которые должны ответить, чтобы разгадать определенный кроссворд с абсолютной УВЕРЕННОСТЬЮ. Предположим, мы знаем точные ответы лишь на три вопроса, но при этом сможем найти решение методом догадки. Однако определенные три буквы могут быть частью совершенно другого слова. В то же самое время, каждый дополнительный ответ усилит нашу уверенность. На самом деле, наука, скорее всего, никогда не сможет ответить на все вопросы, но, чем более убедительные фактические данные мы соберем, тем более точной будет наша догадка в отношении окончательного решения.

    Основные положения

    • Лабораторные исследования клеточного материала проводятся, чтобы определить, существует ли механизм, способствующий возникновению пагубных биологических эффектов под воздействием ЭМП. Исследования на животных чрезвычайно важны для определения возможных эффектов для высших организмов, физиология которых в определенной степени схожа с физиологией человека. Эпидемиологические исследования направлены на установление статистической корреляции между воздействием полей и распространенностью определенных неблагоприятных результатов в отношении здоровья у людей.
    • Выявление статистической корреляции между неким фактором и определенным заболеванием не означает, что этот фактор явился причиной заболевания.
    • Отсутствие эффектов для здоровья может означать, что таковых нет; однако, это может означать и то, что эффект просто не выявляется с помощью существующих методов.
    • Результаты различных исследований (цитологических, эпидемиологических и исследований на животных) следует анализировать в совокупности, прежде чем делать выводы о возможных рисках для здоровья от предполагаемой экологической угрозы. Последовательные данные, полученные в результате этих столь разных по характеру исследований, помогут с большей степенью уверенности судить о действительных эффектах.

    Обычные уровни воздействия в домах и в окружающей среде

    Электромагнитные поля дома

    Уровни фонового электромагнитного излучения от передающих или распределительных электросетевых объектов

    Электричество передается на большие расстояния по высоковольтным линиям. Трансформаторы снижают такое высокое напряжение в сети до требуемого уровня для распределения электроэнергии на местах – в домах и на предприятиях. Передающие и распределительные электросетевые объекты, а также бытовая электропроводка и электроприборы создают в домах фоновый уровень электрических и магнитных полей промышленной частоты. Если дома не расположены вблизи линий электропередач (ЛЭП), фоновый уровень может доходить примерно до 0.2 микротесл. Непосредственно под ЛЭП поля гораздо сильнее. Индукция магнитного поля на уровне земли может достигать нескольких микротесл. Уровни электрических полей непосредственно под ЛЭП могут доходить до 10 кВ/м. Однако поля (как электрические, так и магнитные) по мере удаления от ЛЭП ослабевают. На расстоянии 50-100 метров уровни полей, обычно, такие же, как те, которые наблюдаются на удаленных от высоковольтных ЛЭП территориях. К тому же, стены зданий значительно снижают уровни электрических полей в сравнении с уровнями вне домов в той же местности.

    Электробытовые приборы

    Самые сильные электрические поля промышленной частоты в окружающей среде обычно встречаются непосредственно под высоковольтными ЛЭП. Напротив, самые сильные магнитные поля промышленной частоты обычно наблюдаются в непосредственной близости от двигателей и других электроприборов, а также специализированного оборудования, например магнитно-резонансных томографов, используемых для диагностической визуализации в медицине.

    Обычные значения силы электрических полей вблизи бытовых электроприборов (на расстоянии 30 см от них
    (Источник: Федеральное ведомство по радиационной защите, Германия, 1999 г.)

    Электробытовой приборСила электрического поля (В/м)
    Стерео-проигрыватель180
    Утюг120
    Холодильник120
    Миксер100
    Тостер80
    Фен для волос80
    Цветной телевизор60
    Кофейная машина60
    Пылесос50
    Электропечь8
    Лампочка5
      
    Установленное пороговое значение5000

    Многие люди удивляются, когда узнают о существовании магнитных полей самого разного уровня рядом с различными бытовыми приборами. Сила этих полей не зависит от размера, сложности, мощности таких приборов или уровня шума от них. Более того, сила магнитных полей может очень сильно различаться, даже если речь идет о вроде бы похожих приборах. Например, одни фены для волос окружены очень сильным полем, а другие вряд ли вообще создают какое-либо магнитное поле. Такая разница в отношении силы магнитных полей объясняется дизайном изделия.

    В приведенной ниже таблице указаны обычные значения силы поля для ряда электроприборов, широко используемых дома и на рабочем месте. Измерения производились в Германии, при этом во всех приборах использовался ток с частотой 50 Гц. Следует отметить, что фактические уровни воздействия значительно различаются в зависимости от модели прибора и расстояния от него.

    Обычные значения силы магнитных полей вокруг бытовых электроприборов (в зависимости от расстояния от них)

    Электробытовой прибор

    На расстоянии 3 см (микротесла)

    На расстоянии 30 см (микротесла)

    На расстоянии 1 м (микротесла)

    Фен для волос

    6 – 2000

    0. 01 – 7

    0.01 – 0.03

    Электробритва

    15 – 1500

    0.08 – 9

    0.01 – 0.03

    Пылесос

    200 – 800

    2 – 20

    0.13 – 2

    Флюоресцентный осветительный прибор

    40 – 400

    0.5 – 2

    0.02 – 0.25

    Микроволновая печь

    73 – 200

    4 – 8

    0. 25 – 0.6

    Портативный радиоприемник

    16 – 56

    1

    < 0.01

    Электропечь

    1 – 50

    0.15 – 0.5

    0.01 – 0.04

    Стиральная машина

    0.8 – 50

    0.15 – 3

    0.01 – 0.15

    Утюг

    8 – 30

    0.12 – 0.3

    0.01 – 0. 03

    Посудомоечная машина

    3.5 – 20

    0.6 – 3

    0.07 – 0.3

    Компьютер

    0.5 – 30

    < 0.01

     

    Холодильник

    0.5 – 1.7

    0.01 – 0.25

    <0.01

    Цветной телевизор

    2.5 — 50

    0.04 – 2

    0.01 – 0.15

    Для большинства бытовых электроприборов сила магнитного поля на расстоянии 30 см от них значительно ниже установленного для населения порогового значения в 100 микротесл.

    (Источник: Федеральное ведомство по радиационной защите, Германия, 1999 г.). Нормальная дистанция для работы с прибором выделена жирным шрифтом.

    Таблица иллюстрирует две основные мысли: во-первых, сила магнитного поля вокруг всех приборов стремительно уменьшается по мере того, как вы удаляетесь от них; во-вторых, большинство бытовых приборов работает не слишком близко от человека. На расстоянии 30 см уровень магнитные поля вокруг большинства бытовых приборов более чем в 100 раз ниже установленного для обычного населения порогового значения в 100 микротесл при частоте электрического тока в 50 Гц (и 83 микротесл при частоте тока в 60 Гц).

    Телевизоры и компьютерные мониторы

    В основе работы компьютерных мониторов и телевизоров лежат одни и те же принципы. И те и другие продуцируют статические электрические поля и переменные электрические и магнитные поля разных частот. Однако, жидко-кристаллические мониторы некоторых ноутбуков и настольных ПК не создают значительные электрические и магнитные поля. Мониторы современных компьютеров созданы из проводящих материалов, что снижает статическое поле вокруг монитора до уровней, сопоставимых с нормальным фоновым уровнем в доме или на рабочем месте. Если человек работает на правильном расстоянии (30-50 см) от монитора, уровень индукции переменного магнитного поля (промышленной частоты) обычно ниже 0,7 микротесл. Сила переменных электрических полей при работе на том же расстоянии от монитора находится в интервале от менее 1 В/м до 10 В/м.

    Микроволновые печи

    Бытовые микроволновые печи отличаются большой мощностью. Однако, надежный защитный экран снижает возможную утечку микроволнового излучения за пределы печи до практически неопределяемого уровня. Кроме того, уровень утечки стремительно снижается по мере удаления пользователя от печи. Во многих странах существуют промышленные стандарты, конкретно указывающие предельно допустимые уровни утечки для новых печей. Если печь соответствует этим стандартам, она не представляет никакой угрозы для потребителя.

    Переносные телефоны

    Для работы переносных телефонов требуется гораздо менее интенсивное поле, чем для мобильных телефонов. Это связано с тем, что они используются совсем близко от своей базы, а значит, нет необходимости в сильном поле, как это было бы в случае передачи сигнала на большое расстояние. Соответственно, радиочастотные поля вокруг этих телефонов совсем незначительны.

    Электромагнитные поля в окружающей среде

    Радар

    Радары используются для навигации, составления прогноза погоды, в военных целях, а также для выполнения множества других задач. Они посылают пульсирующие микроволновые сигналы. Пиковая мощность сигнала может быть высокой, между тем как средняя мощность может быть низкой. Многие радары вращаются или движутся вверх и вниз, что уменьшает среднее значение плотности мощности поля, которое воздействует на людей вблизи радара. Даже в отношении высокомощных, не вращающихся военных радарных установок действуют ограничения по уровню воздействия: он должен быть ниже установленного порогового значения в местах, доступных для населения.

    Системы безопасности

    Системы защиты от краж в магазинах основаны на использовании специальных датчиков, закрепляемых на товарах, которые считываются электрическими контурами на выходе. Когда покупка осуществлена должным образом, эти датчики снимают или полностью деактивируют. Электромагнитные поля вокруг контуров обычно не превышают рекомендуемые уровни допустимого воздействия. Системы управления доступом, работают по тому же принципу: датчик встроен в брелок для ключей, либо в пропуск. Системы безопасности в библиотеках используют специальные этикетки-датчики, которые деактивируются при выдаче книги читателю и вновь активируются, когда книга возвращается. Металло-детекторы и системы безопасности в аэропортах создают сильное магнитное поле (до 100 микротесл), которое реагирует на металлические предметы. Вблизи рамки детектора сила магнитного поля может приближаться к установленному пороговому уровню, а иногда и превышать его. Тем не менее, это не создает угрозу для здоровья, о чем будет сказано в разделе, посвященном руководящим принципам по допустимым уровням воздействия (см. «Опасны ли уровни воздействия выше установленных пороговых значений?»).

    Электропоезда и трамваи

    Поезда дальнего следования имеют один или несколько моторных отсеков, расположенных в отдельных вагонах. Таким образом, пассажиры испытывают воздействие полей в основном от электричества, подаваемого в поезд. Магнитные поля в пассажирских вагонах поездов дальнего следования могут достигать нескольких сотен микротесл на уровне пола и более низких значений (десятков микротесл) в других местах в купе. Сила электрического поля может достигать 300 В/м. Люди, живущие вблизи железнодорожных путей, могут испытывать воздействие магнитных полей от линий электропроводов над полотном железной дороги, причем сила этих полей, в зависимости от каждой конкретной страны, может быть сопоставима с силой полей вокруг высоковольтных ЛЭП.

    Двигатели и тяговое оборудование поездов и трамваев обычно располагается внизу, под пассажирскими вагонами. На уровне пола интенсивность магнитного поля может достигать десятков микротесл (на тех участках пола, которые находятся прямо над двигателем). Однако, чем выше от пола, тем быстрее уменьшается интенсивность поля, и его воздействие на верхнюю часть туловища пассажиров значительно слабее.

    Телевидение и радио

    Когда вы у себя дома слушаете радио и ищете нужную вам станцию, задавались ли вы когда-нибудь вопросом, что могут означать хорошо знакомые вам сокращения АМ и FM? Радиосигналы могут быть амплитудно-модулированными (АМ) или частотно-модулированными (FM). Все зависит от того, как они переносят информацию. Радиосигналы АМ могут использоваться для вещания на очень большие расстояния, в то время как FM волны охватывают более ограниченные пространства, но при этом обеспечивают звук лучшего качества.

    АМ радиосигналы передаются при помощи сложной системы антенн, которые могут достигать десятков метров в высоту и располагаться в местах, не доступных обычному населению. Уровни воздействия в непосредственной близости от антенн и кабелей питания могут быть высокими, но с ними приходится иметь дело обслуживающему персоналу, а не обычному населению.

    Телевизионные антенны и антенны для FM радиосигналов гораздо меньше по размеру, чем антенны для АМ радиосигналов, и устанавливаются они как система направленных антенн на самом верху высоких башен. Причем башни являются лишь поддерживающей конструкцией. Поскольку уровень воздействия у самого основания таких башен ниже установленных пороговых значений, доступ обычного населения в места, где находятся такие башни, не запрещен. Небольшие ТВ- и радиоантенны местного значения иногда устанавливаются на крышах зданий; в этом случае не исключается необходимость контролировать доступ на крышу.

    Мобильные телефоны и их базовые станции

    Мобильные телефоны дают нам возможность всегда быть на связи с другими людьми. Эти приборы низкой мощности, испускающие и принимающие радиоволновые сигналы от сети стационарных базовых станций малой мощности. Каждая базовая станция мобильной связи обеспечивает охват определенной территории. В зависимости от потока обрабатываемых звонков, базовые станции могут находиться на расстоянии от всего лишь нескольких сотен метров (в крупных городах) до нескольких километров (в сельской местности) друг от друга.

    Базовые станции мобильной связи обычно устанавливают на крыше зданий или башен, на высоте от 15 до 50 метров. Уровни прохождения сигналов от конкретной базовой станции непостоянны и зависят от количества звонков и расстояния, на котором звонящий абонент находится от базовой станции. Антенны излучают очень узкий пучок радиоволн, который далее распространяется почти параллельно земле. Поэтому радиочастотные поля на уровне земли и на территориях, обычно доступных для населения, во много раз ниже уровней, представляющих опасность.

    Рекомендуемые пороговые значения были бы превышены лишь в том случае, если бы человек оказался прямо перед системой антенн на расстоянии одного-двух метров. До того, как мобильные телефоны стали широко использоваться, население в основном испытывало воздействие радиочастотного излучения от радио- и ТВ-станций. Но и сегодня, с появлением мобильных телефонов, башни, на которых расположены базовые станции мобильной связи, сами по себе крайне мало усугубляют общее воздействие на наш организм, поскольку сила сигналов в местах, доступных для населения, обычно такая же или даже ниже, чем сила сигналов от радио- и ТВ-станций, расположенных на значительном удалении от этих мест.

    Однако на самого пользователя мобильного телефона воздействуют радиочастотные поля более высокого уровня, чем те, которые обычно присутствуют в окружающей нас среде. Разговаривая по мобильному телефону, мы держим его очень близко к голове. Именно поэтому, вместо того, чтобы отслеживать эффект нагревания тканей во всем организме, следует определить распределение поглощенной энергии в голове пользователя телефона. В результате сложного компьютерного моделирования и проведения оценок с использованием моделей головы человека, сделан вывод о том, что, по всей видимости, уровень энергии, поглощенной при использовании мобильного телефона, не превышает установленных на сегодня пороговых значений.

    Вызывают обеспокоенность и другие, так называемые «нетермальные» последствия воздействия частот мобильных телефонов. Есть различные предположения в отношении едва заметных эффектов для клеток, которые могут повлиять на развитие раковых заболеваний. Также высказываются гипотезы о возможных эффектах для тканей, раздражаемых под воздействием электричества, и о том, что это может повлиять на функцию мозга и нервных тканей. Тем не менее, все имеющиеся на данный момент фактические данные не подтверждают наличия каких-либо пагубных последствий для здоровья человека от использования мобильных телефонов.

    Магнитные поля в повседневной жизни: действительно ли они такие сильные?

    В последние годы национальными органами власти различных стран были проведены многочисленные оценки для определения уровней ЭМП в среде обитания человека. Ни одно из этих обследований не пришло к выводу о том, что уровни полей могут вызвать неблагоприятные последствия для здоровья.

    Недавно Федеральное ведомство по радиационной защите (Германия) сделало оценку повседневного воздействия магнитных полей с привлечением к обследованию примерно 2 000 человек. Оценка проведена как в отношении представителей ряда профессий, так и обычного населения. Всем участникам обследования были выданы персональные дозиметры для измерения уровней воздействия 24 часа в сутки. Полученные данные различались весьма значительно, но средний уровень в день составлял 0,10 микротесл. Это значение в тысячу раз меньше, чем предельно допустимое значение в 100 микротесл для обычного населения и в 5 тысяч раз ниже, чем предельное допустимое значение в 500 микротесл для людей определенных профессий. Более того, при исследовании воздействия полей на людей, живущих в центральной части городов, было обнаружено, что, с точки зрения воздействия полей, нет существенной разницы между проживанием в сельской и городской местности. Даже уровни воздействия на людей, живущих в непосредственной близости от высоковольтных ЛЭП, лишь незначительно отличаются от средних уровней воздействия на обычное население.

    Основные положения

    • Фоновые уровни ЭМП в доме в основном создаются передающими и распределительными электросетевыми объектами или бытовыми электроприборами.
    • Электроприборы сильно различаются с точки зрения силы генерируемых ими полей. По мере удаления от приборов уровни как электрических, так и магнитных полей стремительно снижаются. В любом случае, уровни полей вокруг бытовых электроприборов обычно гораздо ниже установленных пороговых значений.
    • Уровни электрических и магнитных полей от телевизоров и компьютерных мониторов (при соблюдении пользователем правильной дистанции от них) в сотни тысяч раз ниже установленных пороговых значений.
    • Микроволновые печи, отвечающие стандартам качества, не представляют опасности для здоровья.
    • Пока действуют ограничения в отношении доступа населения непосредственно к радарным установкам, радиоантеннам и базовым станциям мобильной связи, установленные предельные уровни воздействия радиочастотных полей не будут превышены.
    • Пользователи мобильных телефонов испытывают воздействие полей таких уровней, которые значительно превышают любые значения, регистрируемые в обычной среде обитания. Но, по-видимому, даже столь высокие уровни воздействия не приводят к пагубным последствиям для здоровья.
    • Многочисленные обследования подтвердили, что воздействие электромагнитных полей тех уровней, которые наблюдаются в среде обитания человека, очень незначительно.

    Действующие стандарты

     

    Стандарты устанавливаются с целью защиты нашего здоровья. Широко известно о существовании стандартов для многих пищевых добавок, допустимой концентрации химических веществ в воде или концентрации веществ, загрязняющих воздух. Точно так же есть стандарты и в отношении электромагнитных полей, установленные с целью ограничения чрезмерного воздействия ЭМП, существующих в окружающей среде.

    Кто вырабатывает руководящие принципы по допустимым уровням воздействия?

    Страны самостоятельно устанавливают свои национальные стандарты в отношении допустимого воздействия ЭМП. Однако при формировании большинства национальных стандартов за основу были взяты руководящие принципы, разработанные Международной комиссией по защите от неионизирующей радиации (ICNIRP). Эта неправительственная организация, официально признанная ВОЗ, оценивает результаты научных исследований, проведенных по всему миру. На основании углубленного анализа имеющейся литературы ICNIRP разрабатывает руководящие принципы по допустимым уровням воздействия. Руководящие принципы подвергаются регулярному критическому рассмотрению и, по мере необходимости, обновляются.

    Уровни ЭМП изменяются в зависимости от диапазона частот, и эта зависимость носит сложный характер. Перечисление всех значений для каждого стандарта и каждой частоты было бы затруднительно для понимания. Приведенная ниже таблица обобщает в сжатом виде рекомендации в отношении допустимых уровней воздействия в трех случаях, вызывающих особую обеспокоенность населения: воздействия электричества в домах, базовых станций мобильных телефонов и микроволновых печей. Эти руководящие принципы в последний раз обновлялись в апреле 1998 года.

    Резюме руководящих принципов ICNIRP по допустимым уровням воздействия ЭМП


     Промышленная частота, принятая в ЕвропеЧастота базовой станции мобильной связиЧастота микроволновой печи
    Частота50 Гц50 Гц900 МГц1.8 ГГц2.45 ГГц
     Электрическое поле (В/м)Магнитное поле (микротесла)Плотность мощности (Вт/м2)Плотность мощности (Вт/м2)Плотность мощности (Вт/м2)
    Пороговые значения воздействия для обычного населения5 0001004. 5910
    Пороговые значения воздействия для людей определенных профессий10 00050022.545 

    ICNIRP, EMF guidelines [Руководящие принципы по допустимым уровням воздействия ЭМП], Health Physics №74, 494-522 (1998 г.)

    Рекомендуемые пороговые значения воздействия, принятые в некоторых странах бывшего Советского Союза и в западных странах, могут различаться в 100 и даже более раз. В связи с глобализацией торговли и стремительным внедрением телекоммуникаций во всем мире, возникает необходимость установления неких универсальных стандартов. Поскольку в настоящее время многие страны бывшего СССР работают над созданием новых стандартов, ВОЗ недавно объявила о глобальной инициативе по гармонизации рекомендуемых пороговых значений воздействия. Будущие стандарты будут основаны на результатах Международного проекта ВОЗ по электромагнитным полям.

    Что лежит в основе руководящих принципов?

    Важно отметить, что рекомендуемое пороговое значение само по себе не устанавливает четкую границу между тем, что безопасно и тем, что опасно. Не существует такого единого уровня, выше которого воздействие создает угрозу для здоровья. Напротив, потенциальный риск для здоровья человека возрастает постепенно, по мере увеличения уровней воздействия. В руководящих принципах указано, что, согласно имеющимся научным данным, воздействие ЭМП ниже определенного порогового значения не является опасным. Но из этого не следует автоматический вывод, что если воздействие превышает это определенное пороговое значение, оно непременно представляет опасность.

    Тем не менее, для установления ограничений в отношении воздействия ученые, занимающиеся исследованиями, должны определить пороговый уровень, при котором начинают проявляться первые неблагоприятные последствия для здоровья. Поскольку для проведения экспериментов нельзя использовать людей, при составлении руководящих принципов приходится полностью полагаться на результаты опытов на животных. Незначительные изменения форм поведения животных при низких уровнях воздействия зачастую предшествуют более радикальным изменениям показателей здоровья при более высоких уровнях воздействия. Отклонение в поведении – это очень точный индикатор ответной биологической реакции, и оно было выбрано в качестве самого малозаметного неблагоприятного эффекта для здоровья. В руководящих принципах содержится рекомендация не допускать такие уровни воздействия ЭМП, при которых изменение форм поведения становится заметным.

    Такой пороговый уровень воздействия с точки зрения изменения форм поведения не равен пороговому уровню, рекомендуемому в руководящих принципах. ICNIRP использует коэффициент безопасности, равный 10, при установлении допустимых предельных значений воздействия на людей определенных профессий, и коэффициент, равный 50, для расчета рекомендуемых предельных значений для обычного населения. Например, в диапазоне радиочастот и микроволновых частот, максимальные уровни, с которыми вы можете столкнуться в окружающей среде или у себя дома, по меньшей мере, в 50 раз ниже, чем те пороговые значения, при которых у животных проявляется изменение форм поведения.

    Почему для людей определенных профессий установлены менее жесткие ограничения в отношении допустимого уровня воздействия, чем для обычного населения?

    Лица, которые в силу своей профессии вынуждены испытывать воздействие полей – это взрослые люди, привыкшие работать в хорошо знакомых им условиях электромагнитных полей. Они прошли соответствующую подготовку, чтобы понимать потенциальные риски такой работы и принимать соответствующие меры предосторожности. В отличие от них, обычное население – это люди всех возрастов с разным состоянием здоровья. Во многих случаях обычные люди даже не осознают, что на них воздействуют ЭМП. Кроме того, не следует ожидать, что каждый человек будет принимать меры для того, чтобы избежать вредного воздействия полей или свести его к минимуму. Именно по этим причинам для обычного населения приняты более жесткие ограничения в отношении допустимого уровня воздействия, чем для людей, подвергающихся воздействию ЭМП в силу своей профессии.

    Как было упомянуто выше, низкочастотные ЭМП индуцируют токи в организме человека (см. раздел «Краткое описание последствий для здоровья»). Но и различные биохимические реакции в самом организме человека также генерируют токи. Клетки и ткани не смогут распознать индуцированные токи, если они ниже этого фонового уровня. Вот почему в отношении низкочастотных полей в руководящих принципах по допустимым уровням воздействия закреплено, что уровень токов, индуцированных электромагнитными полями, должен быть ниже уровня токов, естественным образом генерируемых в организме человека.

    Основной эффект от радиочастотной энергии состоит в нагревании тканей. Соответственно, пороговые значения воздействия радиочастотных и микроволновых полей установлены таким образом, чтобы предотвратить пагубные последствия для здоровья от локализованного или общего нагревания организма (см. «Краткое описание последствий для здоровья»). Соблюдение руководящих принципов гарантирует, что эффекты нагревания будут достаточно слабыми и, соответственно, неопасными.

    Чего нельзя предусмотреть в руководящих принципах?

    В настоящее время предположения о возможных долговременных неблагоприятных последствиях для здоровья не могут служить основанием для выпуска соответствующих руководящих указаний или стандартов. Если суммировать результаты научных исследований, совокупность всех доказательств не свидетельствует о том, что ЭМП вызывают долговременные пагубные последствия, например, раковые заболевания. Национальные и международные органы устанавливают и обновляют стандарты на основании самых последних научных знаний, чтобы защитить здоровье людей от общеизвестных неблагоприятных эффектов.

    Руководящие принципы создаются в интересах некого «среднего» населения и не могут напрямую отвечать на запросы того меньшинства, которое, возможно, отличается более высокой чувствительностью. Например, руководящие принципы по допустимым уровням загрязнения воздуха не ориентированы на особые потребности людей больных астмой. Точно так же, руководящие принципы в отношении ЭМП не призваны защищать людей от воздействия, связанного с вживляемыми медицинскими электронными приборами, такими как кардиостимуляторы. Вместо этого, такие пациенты должны обращаться за советом по поводу того, как избежать возможного неблагоприятного воздействия, к производителям и врачам, вживляющим прибор.

    Каковы типичные максимальные уровни воздействия в домашних условиях и в окружающей среде?

    Некоторая информация практического характера поможет вам оценить приведенные выше значения уровней воздействия, установленные в международных руководящих принципах. Ниже в таблице вы найдете информацию о наиболее распространенных источниках ЭМП. Все приведенные значения – это максимально допустимые уровни для обычного населения. Уровень воздействия в вашем случае, вероятнее всего, будет гораздо ниже. Чтобы получить более подробную информацию об уровнях полей вокруг отдельных электроприборов, рекомендуем вам ознакомиться с разделом «Обычные уровни воздействия в домах и в окружающей среде».

    ИсточникТипичный максимальный уровень воздействия для обычного населения
     Электрическое поле (В/м)Индукция магнитного поля (микротесла)
    Естественные поля20070 (магнитное поле Земли)
    Мощность, потребляемая от сети в домах, расположенных не вблизи линий электропередач (ЛЭП)1000.2
    Мощность, потребляемая от сети под крупными ЛЭП10 00020
    Электропоезда и трамваи30050
    Телевизоры и компьютерные мониторы (на правильном расстоянии от них)100. 7
     Типичный максимальный уровень воздействия для обычного населения (Вт/м2)
    Теле- и радиопередающие станции0.1
    Базовые станции мобильной связи0.1
    Радары0.2
    Микроволновые печи0.5

    Источник: Европейское региональное бюро ВОЗ

    Как руководящие принципы применяются на практике, и кто контролирует этот процесс?

    Ответственность за проверку уровней полей вокруг ЛЭП, базовых станций мобильной связи и других источников, доступ к которым обычного населения не ограничен, лежит на государственных учреждениях и местных органах власти. Они должны обеспечить соблюдение руководящих принципов.

    Если речь идет об электронных приборах, за соблюдение стандартных ограничений отвечают производители. Однако, как было упомянуто выше, свойства большинства приборов таковы, что излучение от них гораздо ниже даже малозначимых величин воздействия. Кроме того, многие объединения потребителей регулярно проводят тестирование приборов. В случае, когда вы испытываете особую обеспокоенность или тревогу, рекомендуем вам напрямую связаться с производителем или направить запрос в местный орган общественного здравоохранения.

    Опасны ли уровни воздействия выше установленных пороговых значений?

    Совершенно не опасно съесть баночку клубничного джема до истечения срока годности. Но если вы съедите джем позже, производитель не гарантирует вам хорошее качество продукта. Однако обычно даже через несколько недель или месяцев после истечения срока годности употреблять джем в пищу совершенно безопасно. Аналогичным образом, руководящие принципы в отношении ЭМП гарантируют, что в пределах установленных пороговых значений воздействия не возникнет никаких общеизвестных пагубных последствий для здоровья. Значительный коэффициент безопасности использован при установлении того уровня, который, как общепризнано, вызывает пагубные последствия для здоровья. Поэтому даже если вы подверглись воздействию поля, сила которого в несколько раз превосходит определенное пороговое значение, воздействие на вас все равно будет в пределах безопасности.

    В повседневных ситуациях большинство людей не подвергается воздействию ЭМП с превышением установленных пороговых значений. Обычные уровни воздействия гораздо ниже этих значений. Тем не менее, бывают случаи, когда человек подвергается в течение короткого периода времени воздействию, близкому или даже превосходящему рекомендуемые пороговые уровни. Согласно ICNIRP, воздействие радиочастотных и микроволновых полей следует усреднить по времени, чтобы понять кумулятивные эффекты. В руководящих принципах в отношении воздействия таких полей конкретно указано усреднение по времени (шесть минут), и специально упомянуто как допустимое кратковременное воздействие с превышением пороговых значений.

    Напротив, в руководящих принципах в отношении воздействия низкочастотных электрических и магнитных полей нет усреднения по времени. Еще более усложняет картину наличие так называемого «фактора сопряжения». Под этим понимается взаимовлияние электрических и магнитных полей и тела, испытывающего их воздействие. Фактор сопряжения зависит от размера и формы тела, типа тканей и расположения тела в пространстве по отношению к полю. Руководящие принципы обязаны быть консервативными: ICNIRP всегда исходит из того, что между полем и человеком, испытывающим его воздействие, есть максимальное сопряжение. Поэтому рекомендуемые пороговые уровни обеспечивают максимальную защиту человека. Например, даже если уровни магнитного поля фена для сушки волос или электробритвы оказываются выше рекомендуемых значений, очень слабое сопряжение между полем и головой предотвращает индуцирование электрических токов, которые могли бы превысить установленные предельно допустимые уровни.

    Основные положения

    • Выпускаемые ICNIRP руководящие принципы основаны на современных научных знаниях. Большинство стран используют это международное руководство для формирования своих национальных стандартов.
    • Стандарты в отношении низкочастотных ЭМП предусматривают, что уровень индуцированных токов должен быть ниже обычного уровня фоновых токов в организме человека. Стандарты для радиочастотных и микроволновых полей установлены на таком уровне, чтобы не допустить неблагоприятных последствий для здоровья от локализованного или общего нагревания организма.
    • Руководящие принципы не предусматривают защиту от возможного воздействия медицинских электроприборов.
    • Максимальные уровни воздействия в повседневной жизни обычно гораздо ниже рекомендуемых пороговых значений.
    • Благодаря значительному коэффициенту безопасности воздействие, превышающее установленные пороговые значения, необязательно окажется вредным для здоровья. Кроме того, усреднение по времени в отношении высокочастотных полей и допущение о максимальном сопряжении для низкочастотных полей еще более расширяют границы безопасности.

     

     

    Меры предосторожности

    По мере поступления все новых данных научных исследований вероятность того, что воздействие ЭМП представляет серьезную угрозу для здоровья, уменьшается. Однако определенная неуверенность сохраняется. Некогда чисто научная дискуссия о том, как следует интерпретировать противоречивые данные, превратилась в обсуждение этого вопроса как важной общественной и политической проблемы.

    Публичное обсуждение ЭМП сосредоточено на вопросах потенциального вреда таких полей и часто оставляет без внимания ту пользу, которая связана с технологическим использованием ЭМП. Без электричества наша жизнь замрет. Точно так же теле- и радиовещание стали очевидным фактом современной жизни. Крайне важно анализировать соотношение ценности и потенциальных угроз.

    Охрана общественного здоровья

    Международные руководящие принципы и национальные стандарты по безопасным уровням воздействия электромагнитных полей разрабатываются на основе современных научных знаний и призваны гарантировать, что те поля, с которыми приходится сталкиваться людям, не причинят вреда их здоровью. Чтобы учесть наличие некоторых неопределенностей в знаниях (например, по причине допущенных в экспериментах ошибок, экстраполяции данных с животных на человека или из-за статистической погрешности), при установлении пороговых значений допустимого воздействия используются значительные коэффициенты безопасности.

    Руководящие принципы подвергаются регулярному критическому рассмотрению и, по мере необходимости, обновляются. С учетом существующих неопределенностей, соблюдение дополнительных мер предосторожности рекомендовано в качестве эффективного подхода, который можно взять на вооружение до тех пор, пока наука не пополнит наши знания о влиянии ЭМП на здоровье. Однако выбор конкретного подхода в отношении соблюдения предосторожности и степень его использования чрезвычайно зависит от силы доказательств наличия риска для здоровья, а также масштабов и характера возможных последствий. Меры предосторожности должны быть пропорциональны потенциальному риску.

    Был разработан ряд стратегий по популяризации мер предосторожности в целях привлечения внимания к проблемам, вызывающим обеспокоенность в отношении общественного здоровья, гигиены труда и окружающей среды, а также безопасности в связи с химическими и физическими факторами риска.

    Что рекомендуется делать, пока научные исследования в этой области продолжаются?

    Одна из целей Международного проекта по ЭМП состоит в том, чтобы помочь национальным органам управления взвесить преимущества от использования технологий, основанных на ЭМП, и сопоставить их с возможным обнаружением риска для здоровья от их использования. Помимо этого ВОЗ выпустит рекомендации в отношении мер защиты, если в них возникнет необходимость. На завершение необходимых научных исследований, оценку их результатов и публикацию уйдет несколько лет. Тем временем Всемирная организация здравоохранения подготовила серию указанных ниже рекомендаций:

    • Строго соблюдать существующие национальные и международные стандарты безопасности: такие стандарты, основанные на современных знаниях, разрабатываются для защиты каждого человека с использованием значительного коэффициента безопасности.
    • Соблюдать простые меры защиты: заграждения, установленные вокруг источников сильных ЭМП, позволяют ограничить несанкционированный доступ на территории, где допустимые пороговые значения могут быть превышены.
    • Проводить консультации с местными органами власти и представителями общественности в отношении выбора места строительства новых ЛЭП и базовых станций мобильной связи: нередко при принятии решений о месте строительства требуется учитывать эстетические факторы и особенности восприятия ситуации общественностью. Открытый обмен информацией на стадии планирования может содействовать лучшему взаимопониманию и широкому одобрению строительства нового объекта.
    • Обмениваться информацией: эффективная система информации в области здравоохранения и обмен информацией между учеными, государственными органами, представителями промышленности и общественности может способствовать повышению уровня общей осведомленности о программах, связанных с воздействием ЭМП, и уменьшению недоверия и страхов.

     

    Движущиеся заряды в магнитном поле: сила, отклонение

    Когда мы строим электрические цепи, никогда не стоит использовать рядом с ними магниты. Причина этого в том, что основными единицами электрического поля являются электрические заряды, на которые воздействуют магнитные поля. Давайте посмотрим на электромагнитное влияние на электрический заряд, чтобы увидеть, что происходит, когда мы устанавливаем электрическое поле равным нулю.

    Магнитные поля и электрические заряды

    Современная физика основана на использовании полей, которые являются зависящими от времени физическими объектами, распространяющимися в пространстве. Для электрических явлений мы используем электрические поля и законы, управляющие их поведением, а для магнитных явлений мы используем магнитные поля и законы, управляющие их поведением.

    Важно отметить, что электрические и магнитные поля не являются независимыми друг от друга. Исторически было трудно понять, что оба физических поля являются частью одного общего описания, основанного на зарядах, которые, если они статичны, генерируют только электрическое поле, но при движении также генерируют магнитное поле.

    Здесь нам нужно только рассматривать магнитное поле B как зависящее от времени и пространства векторное поле. Магнитные поля измеряются в Теслах (Тл). Мы также будем рассматривать только точечные частицы с определенным значением заряда q, измеряемым в кулонах (Кл).

    Произведения векторов

    Произведение векторов — это операция между двумя векторами, которая дает другой вектор. Результирующий вектор перпендикулярен двум перемноженным векторам и имеет модуль, который можно вычислить как:

    Здесь, | | указывает модуль вектора, а угол — это угол, образованный между векторами. Векторное произведение обладает следующим свойством: изменение порядка векторов в векторном произведении равно глобальному знаку минус, т. е.:

    Основным следствием рассмотрения векторных произведений является то, что результирующий вектор перпендикулярен плоскости, определяемой двумя другими векторами, и что если их угол равен нулю или 180º, векторное произведение является нулевым вектором.

    Полезным способом определения направления результирующего вектора является использование правила правой руки, показанного на рисунке ниже.

    Рис. 1. Правило правой руки. Acdx, Wikimedia Commons (CC BY-SA 3.0).

    Движущиеся заряды в магнитном поле

    Общий закон, управляющий поведением электрического заряда в присутствии электромагнитного поля, известен как сила Лоренца. В общее выражение входит и влияние внешнего электрического поля, но здесь мы ограничимся ситуациями, когда присутствует только магнитное поле.

    Выражение для силы, действующей со стороны магнитного поля на движущийся электрический заряд:

    Здесь v — векторная скорость, а произведение скорости на магнитное поле — векторное произведение.

    Векторное произведение означает, что сила, действующая со стороны магнитного поля на движущийся заряд, перпендикулярна направлению поля и скорости заряда. Это также означает, что заряды, которые не движутся, не «видят» магнитное поле, поскольку оно на них не действует. Кроме того, если заряд движется в том же направлении, что и магнитное поле, он не почувствует его влияния.

    Заряды, движущиеся в однородном магнитном поле

    Используя математический аппарат предыдущего раздела, мы можем дать феноменологическое описание того, что происходит, когда электрический заряд движется в области, где есть магнитное поле. По формуле силы Лоренца мы можем изучать динамические траектории, а также энергию частиц.

    Математическое описание

    Ограничимся теперь случаем, когда магнитное поле имеет постоянное фиксированное значение B, не зависящее ни от пространства, ни от времени. Ограничимся также случаем постоянной начальной скорости v.

    Наша постановка следующая: точечная частица с зарядом q движется в фиксированном направлении с постоянной скоростью. Без ограничения общности мы можем считать это направление осью x. Частица движется в области, где нет магнитного поля, пока внезапно не включается. Мы будем считать, что магнитное поле перпендикулярно скорости, поэтому у нас есть максимальный вектор от векторного произведения (с синусоидальной функцией, равной единице).

    Рис. 2. Заряды противоположных знаков, приближающиеся к области с магнитным полем, входящим в страницу., Wikimedia Commons

    Как только магнитное поле включается, магнитная сила заставляет частицу повернуться в направлении, определяемом лоренцевской сила. При этом, согласно формуле, указательный палец указывает в направлении движения заряда, а средний — в направлении магнитного поля. Поскольку скорость изменяется из-за действия этой силы, сила теперь действует в другом направлении. Если вы медленно повернете пальцы по правилу правой руки, то поймете, что частица обязана описывать окружность, так как направление силы постоянно меняется.

    Для такого рода установок существует соглашение о направлении магнитного поля, в соответствии с которым мы используем крестики для обозначения магнитного поля, входящего в страницу, и кружки для обозначения магнитного поля, которое выходит из нее, будучи направленным к наблюдателю.

    Рисунок 3. Траектория заряда в магнитном поле, входящего на страницу Wikimedia Commons

    Мы видели, что движущиеся заряды в однородном магнитном поле описывают круговые траектории. Общая теория кругового движения утверждает, что скорость описывающего его объекта не изменяется, а изменяется его скорость (направление), что и происходит с силой Лоренца.

    Это влияет на энергию частицы, поскольку кинетическая энергия пропорциональна квадрату скорости. При постоянной скорости магнитное поле не изменяет энергию. Это требует тщательного рассмотрения при изучении того, как магниты притягивают металлы, поскольку при этом меняется энергия.

    Что такое циклотроны?

    Наконец, мы рассмотрим применение только что изученного эффекта: циклотроны, ускорители частиц, основанные на силе Лоренца.

    По сути, частицы сначала ускоряются благодаря электрическому полю (по прямой линии), а затем попадают в область, где есть магнитное поле, которое заставляет их описывать круговое движение. Интенсивность магнитного поля можно изменить, чтобы воздействовать на частицу с большей силой и изменить ее скорость и скорость. Это позволяет ускорять частицы в круговом контуре.

    Рис. 4. Схема циклотрона, Wikimedia Commons

    Циклотроны были достижением 20-го века, поскольку до этого использовались только линейные ускорители, которые не позволяли поддерживать ускорение. С другой стороны, когда они достигают скорости, близкой к скорости света, эксперименты предполагают, что мы должны искать более совершенные устройства, которые учитывают как радиационные, так и релятивистские эффекты. Эти усовершенствованные устройства известны как синхротроны, которые используются, например, для производства короткоживущих радиоактивных изотопов.

    Ключевые выводы

    • На движущуюся заряженную частицу действует магнитное поле. И заряд, и движение необходимы для того, чтобы поле проявляло силу.

    • Сила, действующая со стороны магнитного поля на заряженную движущуюся частицу, известна как сила Лоренца. Это перпендикулярно направлению движения частицы и магнитному полю.

    • Движущаяся заряженная частица в области с однородным магнитным полем движется по круговой траектории. Однако его скорость и энергия остаются неизменными.

    • Циклотроны и синхротроны — это ускорители частиц, основанные на силе Лоренца.

    электромагнитное поле | физика | Британика

    • Развлечения и поп-культура
    • География и путешествия
    • Здоровье и медицина
    • Образ жизни и социальные вопросы
    • Литература
    • Философия и религия
    • Политика, право и правительство
    • Наука
    • Спорт и отдых
    • Технология
    • Изобразительное искусство
    • Всемирная история
    • В этот день в истории
    • Викторины
    • Подкасты
    • Словарь
    • Биографии
    • Резюме
    • главных вопросов
    • Обзор недели
    • Инфографика
    • Демистификация
    • Списки
    • #WTFact
    • Товарищи
    • Галереи изображений
    • Прожектор
    • Форум
    • Один хороший факт
    • Развлечения и поп-культура
    • География и путешествия
    • Здоровье и медицина
    • Образ жизни и социальные вопросы
    • Литература
    • Философия и религия
    • Политика, право и правительство
    • Наука
    • Спорт и отдых
    • Технология
    • Изобразительное искусство
    • Всемирная история
    • Британика Классика
      Посмотрите эти ретро-видео из архивов Британской энциклопедии.
    • Demystified Videos
      В Demystified у Britannica есть все ответы на ваши животрепещущие вопросы.
    • #WTFact Видео
      В #WTFact Britannica делится некоторыми из самых странных фактов, которые мы можем найти.
    • На этот раз в истории
      В этих видеороликах узнайте, что произошло в этом месяце (или любом другом месяце!) в истории.
    • Britannica объясняет
      В этих видеороликах Britannica объясняет различные темы и отвечает на часто задаваемые вопросы.
    • Руководство по покупке
      Консультация эксперта по покупке. От техники до товаров для дома и здоровья.
    • Студенческий портал
      Britannica — лучший ресурс для учащихся по ключевым школьным предметам, таким как история, государственное управление, литература и т. д.
    • Портал COVID-19
      Хотя этот глобальный кризис в области здравоохранения продолжает развиваться, может быть полезно обратиться к прошлым пандемиям, чтобы лучше понять, как реагировать сегодня.
    • 100 женщин
      Britannica празднует столетие Девятнадцатой поправки, выделяя суфражисток и политиков, творящих историю.
    • Britannica Beyond
      Мы создали новое место, где вопросы находятся в центре обучения. Вперед, продолжать. Спросить. Мы не будем возражать.
    • Спасение Земли
      Британника представляет список дел Земли на 21 век. Узнайте об основных экологических проблемах, стоящих перед нашей планетой, и о том, что с ними можно сделать!
    • SpaceNext50
      Britannica представляет SpaceNext50. От полета на Луну до управления космосом — мы изучаем широкий спектр тем, которые питают наше любопытство к космосу!
    • Введение

    Краткие факты

    • Связанный контент
    • викторины

    СМИ

    • Видео

    Почему движущиеся заряды создают магнитное поле? Объяснение физики – глубокая физика

    Когда я впервые узнал об электромагнетизме, меня учили, что магнитные поля всегда создаются движущимися зарядами , но мне никогда не было ясно, почему движущиеся заряженные частицы специально создают магнитное поле?

    Движущиеся заряженные частицы создают магнитное поле, потому что существует относительное движение между зарядом и тем, кто наблюдает за зарядом. Из-за этого относительного движения заряженная частица создает вокруг себя магнитное поле, что объясняется специальной теорией относительности и тензором электромагнитного поля .

    В этой статье я объясню все это очень подробно и максимально интуитивно.

    Мы рассмотрим, как специальная теория относительности и понятие электромагнитное тензорное поле объясняют, как движущиеся заряды создают магнитные поля.

    Мы также обсудим некоторые распространенные заблуждения, связанные с этим явлением (которых существует множество), чтобы вы на самом деле остались с верной информацией.

    Прочитав эту статью, вы получите глубокое понимание некоторых из наиболее фундаментальных аспектов, связанных с электромагнетизмом.

    Однако, прежде чем мы углубимся в специальную теорию относительности и все такое прочее, давайте пройдемся ровно по что происходит, когда заряд движется и как вообще возникает это явление создания магнитного поля.

    Поскольку в этой статье будут обсуждаться некоторые темы из специальной теории относительности, вы также можете найти мою статью Специальная теория относительности для чайников полезной, так как она охватывает многие вещи, обсуждаемые здесь.

    Содержание

    Создает ли движущийся заряд электрическое и магнитное поле?

    Часто говорят, что если частица заряжена, она создает электрическое поле, а движущийся заряд создает магнитное поле. Итак, создает ли движущаяся заряженная частица оба этих поля?

    Движущаяся заряженная частица создает как электрическое, так и магнитное поле. Это связано с тем, что заряженная частица всегда будет создавать электрическое поле, но если частица также движется, она будет создавать магнитное поле в дополнение к своему электрическому полю .

    С математической точки зрения электрическое поле, создаваемое движущимся зарядом, точно такое же, как поле неподвижного заряда (по крайней мере, при достаточно малых скоростях и отсутствии ускорения).

    Другими словами, электрическое поле в точке зависит только от , обратный квадрату расстояния (r) до заряда: здесь r со шляпой представляет собой единичный вектор, указывающий от заряда к направлению, в котором измеряется поле.

    Однако в дополнение к электрическому полю также создается магнитное поле, если заряд движется .

    Это магнитное поле также зависит от обратного квадрата расстояния (r) до заряда, а также от скорости заряда (v):

    Если эта формула вам не знакома, не волнуйся; математические детали здесь не слишком важны.

    Дело здесь в том, что это магнитное поле (из-за перекрестного произведения) всегда перпендикулярно как направлению, в котором движется заряд, так и направлению электрического поля :

    Формула, приведенная выше, также указывает, что магнитное поле создается только в том случае, если заряд движется . Но вы могли бы спросить, возможно ли, чтобы стационарный заряд также создавал магнитное поле.

    Подсказка: Продвинутый электромагнетизм (и физика в целом) сильно зависит от области математики, известной как 9.0027 векторное исчисление . Если вы новичок в векторном исчислении, идеальной отправной точкой для вас может стать мой новый онлайн-курс «Векторное исчисление для физики» (нажмите, чтобы проверить).

    Цель этого курса — дать вам как математические инструменты , так и интуитивное понимание того, как и почему различные математические концепции используются для описания физики электромагнетизма, классической механики, теории относительности и т. д. . Этот курс займет у вас от полного новичка в векторном исчислении до человека с гораздо более глубоким пониманием , чем у большинства людей.

    Может ли стационарный заряд создавать магнитное поле?

    Неподвижный заряд не создает магнитного поля, его создает только движущийся заряд. Это связано с тем, что для стационарного заряда его электромагнитное поле состоит только из электрического поля, а не из магнитного поля. Это можно понять из свойств тензора электромагнитного поля.

    Ключевым моментом здесь является то, что, согласно классической электродинамике, магнитное поле может быть создано одним из двух явлений:

    • Движущиеся электрические заряды , такие как ток в проводе или просто одна движущаяся заряженная частица.
    • Изменение электрических полей , например, в случае электромагнитной волны (для распространения которой не требуется исходный заряд).

    В случае стационарного заряда ни одно из этих явлений не происходит, поэтому стационарный заряд не создает магнитного поля .

    В принципе, магнитное поле также может быть создано собственным спином заряженной частицы, но это совершенно другое явление, требующее правильного описания квантовой механики.

    Прежде чем мы подробно рассмотрим, почему магнитное поле создается только движущимся зарядом, я хочу выделить здесь ключевой момент; если заряд кажется неподвижным, магнитное поле не создается, а если кажется, что заряд движется, создается магнитное поле .

    Чтобы точно понять, что я имею в виду под словом «появляется», нам нужно взглянуть на специальную теорию относительности и преобразований Лоренца .

    Как специальная теория относительности объясняет магнитное поле движущегося заряда

    В соответствии со специальной теорией относительности электрическое поле в одной системе отсчета может проявляться как магнитное поле в другой системе отсчета (хотя здесь есть и немало тонкостей). это утверждение).

    Теперь, чтобы понять это и, в частности, как именно это связано с тем, почему магнитные поля создаются только движущимися зарядами, нам нужно обсудить понятия Преобразования Лоренца и системы отсчета .

    Преобразования Лоренца: интуитивное объяснение

    Когда мы сравниваем измерения или физические явления между разными наблюдателями, которые могут двигаться относительно друг друга, совершенно ясно, что такие вещи, как пространственные координаты, могут измеряться по-разному.

    Это просто потому, что по-разному движущиеся наблюдатели всегда описывают измерения из своей собственной системы отсчета , которую вы можете представить как систему координат (с осями пространства и времени), привязанную к этому наблюдателю.

    В обычной ньютоновской физике координаты, измеренные в одной системе отсчета, связаны с координатами в другой системе отсчета, движущейся с относительной скоростью к первой, посредством так называемых преобразований Галилея (в одном измерении):

    Все это говорит о том, что наблюдатель, движущийся со скоростью v, будет измерять любую координату x как имеющую значение «vt» (скорость, умноженная на время) меньше, чем неподвижный наблюдатель, и что они оба измеряют время как абсолютно одинаковое.

    Визуально выполнение преобразования Галилея соответствует простому «скольжению» оси времени таким образом, чтобы значения оставались прежними:

    Однако в специальной теории относительности все обстоит иначе. Вместо преобразований Галилея у нас есть преобразований Лоренца , которые выглядят немного сложнее:

    Их можно визуализировать как своего рода вращения-растяжения, в которых оси пространства и времени смешиваются более сложным образом:

    Сейчас, Преобразования Лоренца НА САМОМ ДЕЛЕ физически верны, если принять во внимание специальную теорию относительности , а преобразования Галилея верны лишь приблизительно для медленных скоростей.

    Причина этого действительно исходит из экспериментальных доказательств того, что скорость света всегда постоянна (преобразования Лоренца гарантируют это). Подробнее об этом можно прочитать в моей вводной статье по специальной теории относительности.

    В преобразованиях Лоренца важно то, что координаты времени и пространства смешиваются вместе , что приводит к всевозможным релятивистским явлениям, таким как замедление времени и сокращение длины .

    Однако более глубокая причина этого заключается в том, что в преобразованиях Лоренца смешиваются не только пространство и время, другие физические величины, такие как энергия и импульс, также .

    Оказывается, именно этот эффект также объясняет почему магнитные поля возникают только у движущихся зарядов .

    Более того, поскольку в специальной теории относительности время и пространство могут «смешиваться», их удобно описывать не как отдельные вещи, а просто как разные проявления (компоненты) одной и той же вещи; пространство-время .

    Аналогичная концепция оказывается справедливой и для электрического и магнитного полей.

    Поскольку они могут «смешиваться» вместе при преобразованиях Лоренца, лучше думать об электрическом и магнитном полях как о разных компонентах одного и того же; электромагнитное поле , которое описывается тензорным полем . Я объясню, как именно это работает, позже.

    Как электрическое поле «выглядит» как магнитное поле в специальной теории относительности

    Когда мы выполняем преобразование Лоренца из неподвижной системы отсчета в подвижную систему отсчета, электрическое поле в неподвижной системе отсчета обычно не будет таким же, как электрическое поле в подвижной системе отсчета .

    Что еще более интересно, так это то, что электрическое поле в неподвижной системе отсчета может на самом деле «превратиться» в магнитное поле или в смесь электрического и магнитного полей, если смотреть на нее из движущейся системы отсчета.

    Слово «превратиться в» не лучший способ описать это, но об этом позже в этой статье.

    Физически это означает, что если у нас есть два наблюдателя, один из которых неподвижен, а другой движется относительно другого, неподвижный наблюдатель может видеть только электрическое поле, но движущийся наблюдатель может также видеть магнитное поле .

    Давайте рассмотрим небольшой пример, чтобы проиллюстрировать это. Скажем, у нас есть стационарная заряженная частица, которая создает электрическое поле только в направлении y (и не создает магнитное поле): Здесь y-шляпа представляет собой единичный вектор в направлении y.

    Если затем выполнить преобразование Лоренца в направлении x (мы смотрим на ситуацию из системы отсчета человека, движущегося вдоль оси x), то окажется, что часть исходной y-компоненты электрического поля теперь проявляется как магнитное поле в направлении z.

    Точная формула для этого «нового» магнитного поля дается формулой (вывод этой формулы вы найдете позже, когда мы будем обсуждать тензор электромагнитного поля): Здесь E y — «исходная» y-компонента электрическое поле, v — скорость движущегося наблюдателя, c — скорость света (постоянная).

    В более общем виде эту формулу можно записать так:

    Другими словами, магнитное поле движущейся заряженной частицы каким-то образом получается из преобразований Лоренца (опять же, точный вывод будет сделан позже в этой статье).

    Важная часть этого заключается в том, что фактическая физика, видимая из движущейся системы отсчета, на самом деле ничем не отличается от неподвижной системы отсчета, просто то, как физические явления проявляются по-другому.

    Это потому, что хотя в движущейся системе отсчета появляется «новое» магнитное поле, «исходное» электрическое поле также выглядит иначе, чем в движущейся системе, но общий эффект электромагнитного поля остается прежним .

    Это можно понять, сравнив, например, эффекты «новых» и «первоначальных» электромагнитных сил и увидев, что они все еще совершенно одинаковы в обеих системах отсчета (это я также покажу позже).

      Примечание : приведенные выше формулы являются примерами вещей, которые вы узнаете при изучении векторного исчисления, которое в значительной степени является основой для всего электромагнетизма .0028 . Поэтому, если вы заинтересованы в более глубоком понимании электромагнетизма, я рекомендую ознакомиться с моей книгой  «Векторное исчисление для физики: полный курс самообучения» .  Вы узнаете сложные вещи, например, как  теорема разложения Гельмгольца  объясняет, почему  уравнения Максвелла  имеют именно ту форму, которую они имеют, а также много других интересных вещей. 

    Из всего этого следует вывод, что электрических и магнитных полей НЕ являются фундаментальными объектами в том смысле, что то, что для кого-то выглядит как электрическое поле, для кого-то может показаться магнитным полем.

    Однако реальная физика, которую видят все, остается одной и той же, просто она проявляется по-разному для разных наблюдателей.

    Настоящее объяснение всего этого заключается в том, что вместо того, чтобы рассматривать электрическое и магнитное поля как отдельные объекты, которые случайно превращаются друг в друга во время преобразований Лоренца, мы должны рассматривать их как части одного фундаментального объекта, электромагнитное поле .

    Электромагнитное поле состоит как из электрических , так и из магнитных «частей», и эти части могут быть разными, если смотреть с разных кадров, но «суммарное» электромагнитное поле остается одним и тем же.

    Другими словами, то, что в одной системе отсчета кажется чисто электрическим полем, в другой подвижной системе отсчета выглядит как «смесь» электрических и магнитных полей.

    Ключевая идея здесь в том, что магнитное поле может появиться у движущегося наблюдателя, но НЕ у неподвижного наблюдателя (как мы увидим более подробно позже).

    Это, по-видимому, также предполагает, что магнитное поле является «частью» электромагнитного поля, которая появляется только для движущихся наблюдателей .

    Таким образом, мы наконец можем понять, почему движущийся заряд создает магнитное поле; если заряд неподвижен, то он создает только электрическое поле, но при просмотре заряда из кадра, движущегося относительно заряда, также создается магнитное поле .

    Ключевым моментом здесь является понимание того, что для наблюдателя, наблюдающего за заряженной частицей из движущейся системы отсчета, это точно то же, что и заряд, движущийся относительно наблюдателя .

    Следовательно, магнитное поле появится только в том случае, если существует относительная скорость между заряженной частицей и тем, кто смотрит на заряд. Другими словами, магнитное поле создается только тогда, когда заряженная частица движется .

    Магнитное поле движущегося заряда обусловлено теорией относительности?

    Если электрическое поле, согласно специальной теории относительности, может «выглядеть» как магнитное поле для другого наблюдателя, у вас может возникнуть вопрос, действительно ли сам магнетизм вызван теорией относительности.

    Короче говоря, магнетизм не вызван относительностью. Магнетизм — это самостоятельный феномен, который существует независимо от теории относительности. Однако, согласно специальной теории относительности, разные наблюдатели могут расходиться во мнениях относительно того, является ли данное электромагнитное явление результатом действия магнетизма или электричества .

    Если вы когда-либо сталкивались с обсуждением связи между специальной теорией относительности и магнетизмом, вы могли видеть странный пример того, как электрическое поле заряженного провода каким-то образом «превращается» в магнитное поле, если смотреть из движущейся системы отсчета.

    Однако было бы огромным заблуждением полагать, что само магнитное поле будет вызвано электрическим полем.

    На самом деле электричество НЕ вызывает магнетизм . Это два разных явления, которые не являются причиной друг друга, но специальная теория относительности говорит вам, что два наблюдателя могут расходиться во мнениях относительно того, «выглядит» поле электрическим или магнитным .

    Вопрос не в том, какое из них, электрическое или магнитное, является более фундаментальным или какое из них «вызывает» другое. Скорее, это вопрос как наблюдают эти поля и как они появляются в разных кадрах .

    Действительно, вы должны думать об электрических и магнитных полях как о частях одного фундаментального поля, электромагнитного поля (которое мы подробно обсудим в ближайшее время), и в зависимости от того, кто наблюдает это поле, оно может выглядеть «более электрическим». ” или “более магнитный”.

    Итак, если кто-то скажет вам, что «магнетизм — это просто электричество с применением относительности» или что «магнетизм вызван относительностью», просто знайте, что это чрезмерное упрощение и не совсем верно с фундаментальной точки зрения.

    Теперь, чтобы по-настоящему понять , почему магнитное поле создается только при движении заряда, нам нужно глубже погрузиться в фактическую структуру самих электромагнитных полей и то, как в этом играет роль теория относительности.

    Правильный способ думать об электромагнитных полях

    Большинство людей начинают изучать электромагнетизм, думая об электрическом и магнитном полях как о двух совершенно разных объектах (точнее, как о двух разных векторных полях).

    Однако это не лучший способ думать об этом, если учитывать специальную теорию относительности .

    Вместо того, чтобы думать об электрическом и магнитном полях как о отдельных объектах, мы должны думать о них обоих как о компонентах одного фундаментального объекта; электромагнитное поле .

    Я скоро объясню математические детали электромагнитного поля, но лучше начать с аналогии.

    Представьте, что полное электромагнитное поле похоже на океан; если мы поместим туда неподвижную заряженную частицу и позволим ей колебаться вверх и вниз (оставаясь при этом в одной и той же точке), она создаст эти радиально исходящие круговые волны.

    Вы можете думать об этом как об электрическом поле, создаваемом «стационарным» зарядом:

    Однако, если теперь заряд начнет двигаться в каком-то направлении , эти «волны» имеют тенденцию распространяться перпендикулярно направлению его движения (если вы когда-нибудь видели корабль, движущийся в океане, он создает вокруг себя этот V-образный волновой узор).

    Вы можете думать об этом как об аналоге магнитного поля, создаваемого движущимся зарядом ; они возникают только тогда, когда объект движется, и имеют тенденцию возникать перпендикулярно направлению движения.

    Конечно, эта аналогия с «водяной волной» ни в коем случае не совсем то, что происходит с электрическими и магнитными полями.

    Однако здесь он иллюстрирует основную идею; хотя эти водные волны выглядят по-разному для стационарных и движущихся наблюдателей, они все же являются «частями» основной структуры, которой является сам океан .

    Просто волны по-разному проявляются, если смотреть из движущейся системы, но они все еще океанские волны.

    Теперь, ту же идею следует применить к электрическому и магнитному полям ; фундаментальным физическим объектом здесь является электромагнитное поле , а не отдельные электрические и магнитные поля.

    Эти электрические и магнитные поля проявляются по-разному, если смотреть на них с разных точек зрения (соответствующих разным конфигурациям электрических и магнитных полей), но они по-прежнему являются частями самого электромагнитного поля .

    Итак, чтобы по-настоящему понять весь этот «релятивистский электромагнетизм», мы должны отказаться от размышлений об электрическом и магнитном полях как о отдельных вещах и вместо этого просто думать о них как о различных проявлениях фундаментального «полного» электромагнитного поля.

    В частности, магнитное поле представляет компоненты электромагнитного поля, наблюдаемые при относительном движении между кадрами , именно поэтому кажется, что заряженная частица создает магнитное поле только тогда, когда она движется.

    Тензор электромагнитного поля и магнитное поле движущегося заряда

    Обычно мы рассматриваем электрические и магнитные поля как векторных полей , которые «присваивают» вектор каждой точке пространства.

    Однако при объединении полное электромагнитное поле (которое состоит как из электрического, так и магнитного поля в каждой точке пространства) математически представляет собой тензорное поле , которое «присваивает» тензор каждой точке пространства.

    Это электромагнитное поле описывается тензор электромагнитного поля , который является фундаментальным физическим объектом во всем электромагнетизме. Вы можете думать об этом как о , описывающем электрические и магнитные поля в каждой точке пространства .

    Что такое тензор электромагнитного поля на самом деле? Ну, чтобы ответить на этот вопрос, нам нужно понять, что такое тензор, и для этого нам нужно сначала отметить пару вещей о векторах.

    Прежде всего, если мы рассматриваем вектор из другой системы отсчета, его компоненты, как правило, будут другими, но сам фактический вектор (его длина и направление) не будет 0028 .

    Примечание: эти векторы с «компонентом времени» (v t ) и «компонентом пространства» (v x ) называются четырехвекторами, которые можно рассматривать как векторов в пространстве-времени . Я обсуждаю четыре вектора более подробно в моем введении в специальную теорию относительности, которое можно найти здесь.

    Это свойство называется ковариацией и является одной из определяющих характеристик вектора.

    Другими словами, при выполнении преобразования Лоренца в движущуюся систему отсчета компоненты вектора будут смешиваться вместе (но фактический вектор останется прежним «стрелка»).

    То же самое происходит и с тензором , и именно так тензор часто определяется в физике; тензор — это геометрический объект, компоненты которого могут изменяться при преобразованиях Лоренца, но сам тензор (его геометрические свойства) остается неизменным .

    На самом деле, это свойство тензоров в значительной степени является причиной того, что тензоры используются и в общей теории относительности . Если вам интересно, я объясню это более подробно в моей статье 9.0027 Общая теория относительности для чайников .

    Обычно компоненты вектора представляются в виде «списка» или столбца. В специальной теории относительности мы обычно имеем дело с четырехвекторов , которые имеют как обычные «компоненты пространства», так и «компонент времени»: здесь индекс µ (который может принимать значения 0,1,2,3 ) обозначает, о какой из этих четырехвекторных компонент мы говорим. Например, µ=0 будет относиться к v t , а µ=3 к v z . Это стандартная часть обозначений, используемых в специальной теории относительности.

    Между прочим, это обозначение индекса для векторов, которое вы видите выше, я подробно рассматриваю в своем курсе Vector Calculus For Physics , так что если вы хотите узнать это лучше, ознакомьтесь с курсом! Я также расскажу о таких вещах, как преобразования координат , одним из примеров которых являются преобразования Лоренца.

    Аналогично, тензор (в данном случае тензор 4×4) — это объект, который может быть представлен в виде «таблицы» материалов, являющихся его компонентами тензора :Здесь снова и µ, и ν имеют значения от 0 до 3, поэтому, например, компонент с µ=0 и ν=2 будет представлять T 02 =T ty .

    Как все это связано с электромагнитным полем? Ну, электромагнитное поле представлено тензором электромагнитного поля, который представляет собой тензор 4 × 4 с электрическим и магнитным полями в качестве его компонентов компонентов электрического поля, B — компоненты магнитного поля (какими бы они ни были для любой заданной конфигурации электромагнитного поля), а c — скорость света, т. е. константа. Более того, этот тип тензора ( антисимметричный тензор ) имеет только 6 независимых компонент, соответствующих 3 компонентам электрического поля и 3 компонентам магнитного поля.

    Этот объект в основном описывает любое наблюдаемое нами электромагнитное поле.

    Теперь, компоненты этого тензора поля могут также смешиваться при преобразованиях Лоренца , где мы приходим к упомянутому ранее «смешению» электрического и магнитного полей.

    Я очень скоро покажу, как это происходит математически, но интуитивно, когда мы делаем преобразование Лоренца (смотрим на электромагнитное поле с движущейся точки зрения), компоненты этого тензора поля будут разными и «перемешаются» вместе.

    Физически это соответствует «смешению» электрического и магнитного полей, а это означает, что из движущейся системы отсчета электрическое поле может выглядеть как магнитное поле и наоборот .

    Именно по этой причине магнитное поле появляется только при движении заряда.

    Электромагнитное поле, которое рассматривается как чисто электрическое поле в неподвижной системе отсчета, будет частично отображаться как магнитное поле, если смотреть из движущейся системы отсчета .

    Однако это НЕ означает, что магнитные поля — это просто «электрические поля в движущейся системе отсчета».

    По сути, и электрическое, и магнитное поля представляют собой различных физических полей, которые оба являются компонентами электромагнитного поля и оба являются их собственными объектами.

    Просто так получается, что магнитное поле — это часть электромагнитного поля, возникающая в движущейся системе отсчета, а НЕ то, что само электрическое поле каким-то образом «превращается» в магнитное поле при движении заряда .

    Далее, давайте посмотрим, как именно эти компоненты электромагнитного поля появляются из преобразований Лоренца математически.

    Преобразование Лоренца тензора электромагнитного поля

    То, как магнитные поля математически появляются в движущихся системах отсчета, представляет собой преобразование Лоренца тензора электромагнитного поля.

    В частности, при выполнении преобразования Лоренца из системы отсчета с неподвижным зарядом в систему, в которой теперь кажется, что заряд движется , получаем новые компоненты тензора электромагнитного поля.

    Мы называем это магнитным полем , но с фундаментальной точки зрения это всего лишь компоненты электромагнитного поля.

    Теперь здесь мы рассмотрим случай конфигурации стационарного заряда , которая создает электрическое поле в направлении y . Итак, изначально у нас есть электрическое и магнитное поля вида (здесь представлены в виде этих векторов-столбцов):

    Другими словами, у нас есть «начальное» электромагнитное поле в форме тензора электромагнитного поля :

    Для этого примера мы хотим теперь выполнить преобразование Лоренца в направлении x .

    Физически это означает, что теперь мы смотрим на ситуацию с точки зрения другого наблюдателя, который движется в направлении x (с постоянной скоростью).

    Однако с точки зрения движущегося наблюдателя оказывается, что теперь также магнитное поле в направлении z, в дополнение к электрическому полю в направлении y (которое имеет другое значение в движущейся системе отсчета).

    Ключевым здесь является понимание того, что если смотреть с точки зрения другого (движущегося) наблюдателя, конфигурация заряда теперь выглядит так, как будто он движется в противоположном направлении , в то время как наблюдатель кажется неподвижным (если смотреть со своей собственной точки зрения , что и делает преобразование Лоренца).

    Это «новое» магнитное поле, наблюдаемое с точки зрения движущегося наблюдателя (именно это представляет здесь символ «-»), математически имеет вид:

    Вывод этого приведен ниже.

    Полный тензор электромагнитного поля с движущейся точки зрения теперь имеет следующую форму:

    Другими словами, с точки зрения движущегося наблюдателя (который теперь видит движущуюся конфигурацию заряда) электромагнитное поле конфигурации заряда по-видимому, имеет различных электрических полей в направлении y , а также магнитных полей в направлении z .

    Именно так движущийся заряд «создает» магнитное поле; , если смотреть из системы отсчета, в которой движется заряд, теперь кажется, что электромагнитное поле также имеет магнитную составляющую .

    Ниже я включил полные математические детали этого преобразования Лоренца, обсуждаемые здесь, для тех из вас, кому это интересно.

    Я также включил некоторое обсуждение того, как фактические физические последствия этих двух, казалось бы, разных конфигураций электромагнитного поля на самом деле являются одними и теми же .

    Другими словами, физика, связанная с электромагнитным полем, остается неизменной, независимо от того, из какой (инерциальной) системы отсчета наблюдается поле.

    Преобразование Лоренца электромагнитного поля математически и его физические следствия

    Преобразование Лоренца в направлении x (т.е. в системе отсчета движущегося наблюдателя) может быть представлено в виде матрицы преобразования координат вида: Здесь v — скорость движущегося наблюдателя в направлении x, c — скорость света, γ — фактор Лоренца γ=(1-v 2 2 ) -1/2 . Эти индексы µ и α здесь (которые оба имеют значения от 0 до 3) просто обозначают компоненты этой матрицы, а символ «-» представляет тот факт, что это преобразование в «штрихованную» систему координат движущегося наблюдателя.

    В случае, если это кажется вам незнакомым, я на самом деле рассказываю о преобразованиях координат, о том, как они используются на практике, и обо всем, о чем мы собираемся поговорить здесь, в моем Vector Calculus For Physics: A Complete Self-Study Course (нажмите, чтобы проверить, если вам интересно узнать больше).

    Во всяком случае, правило преобразования Лоренца для тензора электромагнитного поля выглядит следующим образом: α,β -индексы здесь являются просто «фиктивными» индексами, что означает, что они должны суммироваться от 0 до 3. Этот «штрихованный» тензор поля здесь , (F µν )’ — тензор электромагнитного поля (точнее, его компоненты) в подвижной системе отсчета, а F αβ представляет собой компоненты поля в исходной неподвижной системе отсчета.

    Здесь нужно сначала просуммировать по этим α- и β-индексам от 0 до 3: =2) и µ=2, ν=1 (а также µ=1, ν=2). Когда µ=2 и ν=0, эта сумма сводится к следующему:

    . Затем, подставив сюда все различные компоненты (Λ 2 2′ =1, Λ 0 0′ =γ и F 20 =E y /c), мы получаем:

    Это тоже самое, что и -(F 02 )’. Теперь для случая µ=2, ν=1 сумма сводится к:

    Подставляя сюда все компоненты (Λ 2 2′ =1, Λ 1 0′ =-γv/c и F 20 =E y /c), получаем:

    Это тоже самое, что и -(F 12 )’ -компонент. Затем мы можем собрать все компоненты этого «нового» тензора электромагнитного поля в:

    Это электромагнитное поле, как видно из кадра, в котором кажется, что движется заряд. Мы видим, что, хотя «исходное» электрическое поле имело только y-компоненту (E y ), «новое» электрическое поле от движущейся системы отсчета также имеет y-компоненту, но теперь это γE y вместо исходного E y .

    Более того, электромагнитное поле, как его видит движущийся наблюдатель, теперь также имеет z-компоненту магнитного поля (поскольку этот F 21 -слот обычно представляет B z ), которая первоначально была равна нулю. Эта z-компонента магнитного поля определяется выражением: Это vE y -произведение фактически может быть записано как величина векторного произведения между вектором скорости v=(v,0,0) и «исходным» электрическим полем вектор E=(0,E и ,0). В любом случае, это всего лишь дополнительная деталь, и она просто говорит вам, что «новое» магнитное поле направлено перпендикулярно как к электрическому полю, так и к скорости.

    Мы также можем записать эту формулу как:

    Если бы вы добавили сюда электрическое поле точечного заряда, вы бы получили: Здесь я также использовал определение скорости света, c=1/ √µ 0 ε 0 .

    Это действительно стандартная формула для магнитного поля движущейся заряженной частицы (или с малой скоростью, γ≈1). Довольно интересно посмотреть, как это можно вывести из специальной теории относительности.

    Однако самое интересное во всем этом то, что реальная физика остается одинаковой в обеих системах отсчета.

    Мы можем это понять, если посмотрим, как электромагнитные силы, возникающие в этих полях, воздействуют на заряженную частицу в обоих этих полях.

    В частности, рассмотрим изменение импульса частицы (с зарядом q), вызванное, во-первых, исходным электромагнитным полем. Сила, создаваемая первоначальным полем, состоит только из электрической силы, определяемой формулой: y-шляпа здесь представляет собой единичный вектор в y-направлении.

    Итак, если смотреть из стационарной системы отсчета (имеющей только y-компоненту электрического поля), импульс заряженной частицы изменится (за время Δt; для простоты мы предполагаем, что поле постоянно со временем) на величину:

    Однако, если смотреть из движущейся системы отсчета (т. е. той системы отсчета, в которой та же самая заряженная частица с зарядом q, которую мы анализируем, теперь кажется движущейся в противоположном направлении со скоростью -v), существует теперь другое электрическое поле, а также магнитное поле.

    Электромагнитная сила, действующая на заряженную частицу, как видно из этой системы отсчета, теперь будет:

    Эти «штрихованные» поля здесь представляют собой электрические и магнитные поля, как видно из движущейся системы отсчета, а эта «штрихованная» скорость есть скорость которую заряженная частица q имеет в системе движущегося наблюдателя:

    Чтобы лучше проиллюстрировать всю эту ситуацию, вот картина того, что происходит:

    В любом случае, если мы подставим все компоненты вектора в силу, мы получить следующее: Этот базисный вектор y-шляпы во втором члене происходит от перекрестного произведения между v ‘и B ‘.

    Однако этот 1-v 2 /c 2 справедлив: Напоминание: коэффициент Лоренца равен γ=(1-v 2 /c 2 ) -1/2 .

    Таким образом, электромагнитная сила в движущейся системе отсчета равна:

    Теперь наступает важная часть; если смотреть на заряженную частицу q из движущейся системы отсчета, ее время также кажется замедленным из-за замедления времени (я подробнее обсуждаю замедление времени в этой статье).

    Итак, движущийся наблюдатель фактически видит время, прошедшее для заряженной частицы, как: Здесь Δt — это время, прошедшее в системе отсчета самой заряженной частицы, а Δt’ — это время, прошедшее с точки зрения движущегося наблюдателя.

    Таким образом, изменение импульса заряженной частицы за время Δt, , если смотреть из системы отсчета движущегося наблюдателя , будет:

    Это в точности то же самое, что мы имели в неподвижной системе отсчета! Итак, каким-то образом все «релятивистские» эффекты между движущейся и неподвижной системой отсчета приводят к одним и тем же физическим последствиям для заряженной частицы.

    Другими словами, физические результаты электромагнитного поля абсолютно одинаковы для обеих систем отсчета, просто оно проявляется по-разному для разных наблюдателей (например, в виде электрического поля для одного наблюдателя и комбинации электрического и магнитное поле для другого наблюдателя).

    электромагнетизм — Почему магнитные поля действуют на движущиеся свободные заряды?

    Задавать вопрос

    спросил

    Изменено 5 лет, 8 месяцев назад

    Просмотрено 3к раз

    $\begingroup$

    Я могу понять, почему ферромагнетики создают вокруг себя магнитное поле из-за ориентации магнитного вращения их электронов, и как другие постоянные магниты могут реагировать на это магнитное поле, потому что материал намагничен.

    Но почему движущийся заряд отклоняется магнитным полем? Я думаю, что она вовсе не намагничена, и еще более нелогично, что сила, действующая на рассматриваемую частицу, перпендикулярна магнитному полю, в отличие от того, что происходит в электрическом или гравитационном полях.

    Почему свободные заряды и намагниченные объекты ведут себя по-разному в магнитном поле и почему движущиеся свободные заряды вообще чувствуют поле?

    • электромагнетизм
    • магнитные поля

    $\endgroup$

    10

    $\begingroup$

    Магнитное поле B определяется в терминах силы Лоренца, действующей на движущиеся заряженные частицы, так что частица, движущаяся в электромагнитном поле, испытывает силу $q\vec{E} +q\vec{v }\times\vec{B}$ (см. здесь).

    При рассмотрении в контексте специальной теории относительности существует только электромагнитное поле. То, что мы определяем как электрические или магнитные поля, является просто зависимыми от системы отсчета проявлениями этого поля — отсюда и член скорости в силе Лоренца.

    Начав с базовой идеи о том, как электрические поля действуют на заряженные частицы, вы можете продемонстрировать, что требуется магнитная составляющая силы Лоренца, действующая перпендикулярно скорости, используя такой аргумент, который я не буду вырезать и вставлять. здесь.

    $\endgroup$

    $\begingroup$

    Как вы правильно сказали, заряженных частиц имеют магнитный дипольный момент , вы назвали его магнитным спином. Дело в том, что неподвижный электрон под действием магнитного поля выровняется только один раз. Для движущегося заряда его магнитный дипольный момент будет выровнен и за счет гироскопического эффекта электрон будет наклоняться (отклоняться).

    Прогиб — это ускорение и известно, что в этом случае фотонов будет испущено электроном . Так как фотоны переносят импульс, происходит рассогласование магнитного дипольного момента с внешним полем. И игра начинается снова: выравнивание, отклонение, испускание фотонов, разъединение. Электрон теряет кинетическую энергию, траектория представляет собой спиральный путь, и более того, спираль состоит из ломтиков мандарина.

    Действие внешнего магнитного поля подобно действию пружины. Внешнее поле не вносит энергии в отклонения в сумме.

    Из сказанного понятно, почему электрон, движущийся параллельно внешнему магнитному полю, не отклоняется. Простой гироскопический эффект не работает.

    $\endgroup$

    $\begingroup$

    Вы можете найти ответ на свой вопрос в недавней работе, которая объясняет магнитную силу как чисто электростатическую из-за взаимодействия электрических сил между движущимися зарядами. магнитная сила между двумя нитевидными элементами тока». Объяснение было доказано выводом закона магнитной силы и закона Био-Савара с использованием основы электрических сил, как указано в электромагнитной теории для постоянных токов. Объяснение слишком длинное, чтобы описывать его здесь. Вы можете прочитать статью по адресу http://ieeexplore.ieee.org/stamp/stamp.jsp?arnumber=754689.3 . Короче говоря, электрические токи, даже создаваемые движущимися зарядами, эквивалентно представлены зарядами, движущимися со скоростью света. Анализируя электрическое поле, распространяющееся в пространстве этими движущимися зарядами, обнаруживается, что они меняются от положительного к отрицательному и от отрицательного к положительному из-за переключения между движущимися отрицательными и положительными зарядами для создания тока. Применяя закон Гаусса, положительные и отрицательные заряды находятся в местах разрыва электрического поля. Эти заряды взаимодействуют с движущимися токовыми зарядами других элементов, создавая магнитную силу, наблюдаемую благодаря существованию источника тока элемента. Эти разрывные заряды создают нулевую результирующую силу на статический заряд, в то время как на движущийся заряд создают наблюдаемую магнитную силу.

    $\endgroup$

    Производство электромагнитных волн

    Производство электромагнитных волн
    • заряженная частица производит электрическое поле. Это электрическое поле действует на другие заряженные частицы. Положительные заряды ускоряются в направлении поля а отрицательные заряды ускоряются в направлении, противоположном направлению поле.
    • движущаяся заряженная частица производит магнитное поле. Это магнитное поле воздействует на другие движущиеся обвинения. Сила, действующая на эти заряды, всегда перпендикулярна направление их скорости и, следовательно, меняет только направление скорость, а не скорость.
    • Ускорение заряженной частицы производит электромагнитную (ЭМ) волну. Электромагнитные волны являются электрическими и магнитные поля, движущиеся через пустое пространство со скоростью света c. Заряженная частица, совершающая колебания около положения равновесия, представляет собой ускоряющая заряженную частицу. Если его частота колебаний равна f, затем он производит электромагнитную волну с частотой f. длина волны λ этой волны определяется выражением λ = c/f. Электромагнитные волны транспортировать энергию через пространство. Эта энергия может передаваться заряженным частиц на большом расстоянии от источника.

    Ускоряющие заряды создают изменяющиеся электрические и магнитные поля. Изменяющиеся электрические поля производят магнитные поля, а меняющиеся магнитные поля производят электрические поля.   Это взаимодействие между индуцированными электрическими и магнитными полями приводит к распространяющиеся электромагнитные волны. электромагнитный волны могут распространяться в свободном пространстве.

    Предположим, что заряд q, расположенный вблизи начала координат, ускоряется. Поэтому он производит электромагнитное излучение. В какой-то позиции r в пространстве и в некоторый момент времени t, электрическое поле электромагнитной волны, создаваемое ускоряющим зарядом дается выражением

    E
    рад ( r ,t) = -[1/(4πε 0 )]*[q/(c 2 r’)]* 9 на 8 p (т — р’/с).

    Проанализируем это выражение. Электрическое поле пропорционально заряд q. Чем больше ускоряющий заряд, тем больше поле. Это уменьшается обратно пропорционально расстоянию r’, которое является расстоянием между ускоряющий заряд и положение, в котором наблюдается поле. Но это не так расстояние в момент наблюдения поля, но расстояние в несколько более раннее время, называемое замедленное время , когда возникло поле излучения. Все электромагнитные волны распространяются с скорость света c = 3*10 8 м/с в свободном пространстве. Им требуется интервал времени ∆t = ∆r/c, чтобы пройти расстояние ∆r. Электрическое поле также пропорциональна ускорению заряда. Чем больше ускорение, тем больше поле. В приведенном выше выражении Е рад ( р ,т) пропорциональна a перп , компонент ускорение перпендикулярно лучу зрения между r и запаздывающее положение заряда. Направление E рад ( r ,т) перпендикулярно этой линии обзора и ее величина пропорциональна составляющей ускорения, перпендикулярной это линия взгляда.

    На рисунке справа показана эта точка. Электрическое поле равна нулю на линии взгляда в направлении ускорения, наибольшая вдоль луча зрения, перпендикулярного направлению ускорения, и всегда перпендикулярно линии взгляда.

    Величина a perp равна a*sinθ, и поэтому величина поля излучения равна
    E рад ( r ,t) = -[1/(4πε 0 )]*[q/(c 2 r)]*sinθ*a( т — р/с).
    Здесь θ — угол между лучом зрения и направлением ускорение.

    Магнитное поле электромагнитной волны перпендикулярно электрическому полю и имеет величину B рад = E рад /c дюйм свободное место. Для электромагнитных волн E и B являются всегда перпендикулярны друг другу и перпендикулярны направлению распространение. Направление распространения – это направление Е × В .

    Поле излучения E рад , создаваемое ускоряющим точечным зарядом, убывает как 1/r, а статическое кулоновское поле убывает как 1/r 2 . Статическое поле убывает с расстоянием гораздо быстрее, чем поле излучения, и поэтому поле излучения будет преобладать на большом расстоянии для ускоряющего заряда дистрибутивы. Кроме того, радиационные поля создаются только ускоряющие заряды (часто электроны), в то время как статические поля создаются всеми зарядами (положительные ядра и отрицательные электроны) и компенсируют друг друга.

    Вдали от источника электромагнитной волны мы часто лечим ЭМ волна как плоская волна. Синусоидальная плоская электромагнитная волна, распространяющаяся в x-направление имеет форму

    E
    (x,t) = E max sin(kx — ωt + φ),
    B (x,t) = B макс sin(kx — ωt + φ).

    Если для волны, бегущей в направлении x, E точек в y-направлении, затем B точек в z-направлении. Электромагнитный волны поперечные волны .

    Волновой вектор k указывает в направлении распространения, и его величина k = 2π/λ, где λ это длина волны. Частота волны f равна f = ω/2π, ω — угловая частота. Скорость любой синусоидальной волны есть произведение ее длина волны и частота.

    v = λf = ω/k.

    Скорость любых электромагнитных волн в свободном пространстве это скорость света c = 3*10 8 РС.
    Электромагнитные волны в свободном пространстве могут иметь любую длину волны λ или частоту f при условии, что λf = c.
    Видимый свет — это любая электромагнитная волна с длиной волны λ между приблизительно 400 нм и 750 нм.

    Проблема:

    Электромагнитная волна в вакууме имеет амплитуду электрического поля Е макс. = 220 В/м. Рассчитайте амплитуду B max соответствующего магнитного поля.

    Решение:

    • Обоснование:
      Величина магнитного поля B = E/c.
    • Детали расчета:
      B макс. = E макс. /c = (220 Н/З)/(3*10 8 м/с) = (7,33*10 -7 Н/Ам) = 7,33*10 -7 Т.
    Проблема:

    Глаз наиболее чувствителен к свету с длиной волны 5,5*10 -7 м, что находится в зелено-желтой области электромагнитного спектр. Какова частота этого света?

    Решение:

    • Обоснование:
      Для всех электромагнитных волн в свободном пространстве λf = c.
    • Детали расчета:
      f = c/λ = (3*10 8 м/с)/(5,5*10 -7 м) = 5,455*10 14 Гц.
    Проблема:

    Плоская электромагнитная волна распространяется через пространство. В некоторой плоскости в некоторый момент времени t поля однородны и ориентирован, как показано. Каково направление распространения плоская волна?

    Решение:

    • Обоснование:
      Направление распространения – это направление Е × В . Используя правило правой руки, находим, что направление распространения находится на странице.

    Электромагнитные волны могут проходить через прозрачные среды, такие как вода и стакан. В среде они взаимодействуют с атомами или молекулами, и за счет это взаимодействие имеет кажущуюся скорость, отличную от c. Когда электромагнитные волны распространяются через среду, скорость волн в среда v = c/n, где n — показатель преломления среды . Когда ЭМ волна распространяется из одной среды с показателем преломления n 1 в другую среду с другим показателем преломления n 2 , то ее частота остается прежней, но изменяется ее скорость, а значит, и длина волны изменения. Для воздуха n почти равно 1, для воды n равно 1,33, а для большинства очки n составляет около 1,5.

    Проблема:

    Какова кажущаяся скорость света в воде?

    Решение:

    • Обоснование:
      Когда электромагнитные волны распространяются через среду, скорость волн в среде v = c/n, где n — показатель преломления среды.
    • Детали расчета:
      v = c/n = (3*10 8 м/с)/1,33 = 2,26*10 8 м/с.

    Радиоволны

    Большинство радиоволн излучается зарядами, колеблющимися в антеннах. направление разгона зарядов вдоль антенны. Радио волна распространяется от антенны к приемнику прямолинейно путь, называемый линией прямой видимости. Направление электрического поля E электромагнитного излучения, испускаемого антенной лежит в плоскости, содержащей антенну и линию прямой видимости на приемник, и перпендикулярно линии взгляда. Волна поляризовано , что означает, что E имеет четко определенное направление.

    Электрическое поле самое сильное и его интенсивность самая высокая в направлениях перпендикулярно антенне и уходит в ноль в направлении вдоль антенны . Очень плохой прием, если стоять под антенной.

    Для передачи информации электромагнитная волна должна быть модулирована. информация, переносимая радиоволнами, является звуком. Амплитуда АМ (амплитуда модулированная) радиоволна представляет собой колебания давления, которые составляют звук. Частота FM (частотно-модулированных) радиоволн может быть сдвинута немного от их номинальной несущей частоты. Количество смен пропорциональны изменениям давления, из которых состоит звук.

    184_notes: Magnetic_field [Проекты и практики по физике]

    Главы 17 и 20 Материи и взаимодействий (4-е издание)

    Магнитное поле

    Когда заряды покоятся, они создают электрическое поле. Приведите этот заряд в движение, и вы получите новое наблюдение: движущихся зарядов создают магнитное поле . Это поле, изначально считавшееся совершенно отличным от электрического поля, имеет свою форму взаимодействия. Как и электрическое поле, магнитное поле также пронизывает все пространство и становится слабее по мере удаления от источника. Магнитное поле, создаваемое некоторым движущимся зарядом, имеет величину и направление в каждой точке пространства. 92}.$$

    где вектор $\vec{r}$ по-прежнему является вектором разделения между местоположением движущегося заряда в данный момент времени и местом наблюдения в то же время — при условии, что заряд движется намного медленнее, чем скорость света. Опять же, мы не можем вывести это уравнение, это модель магнитного поля точечного заряда, которая хорошо соответствует данным/наблюдениям. Подобно электрическому полю точечного заряда, это исходная точка для магнитных полей (а позднее и для электромагнетизма вообще).

    Магнитная сила, действующая на движущийся заряд в магнитном поле

    Если в сцену ввести другой движущийся заряд, он будет взаимодействовать с первым движущимся зарядом через магнитное поле, создаваемое первым зарядом. (Он также будет испытывать электрическую силу, но мы часто сначала ограничиваем наше обсуждение изолированным магнитным взаимодействием.) Толчок или притяжение, которые испытывает новый движущийся заряд, напрямую связаны с перекрестным произведением скорости этого заряда и магнитного поля. поле исходного заряда. Эта магнитная сила просто

    $$\vec{F}_{B} = q_{тест}\vec{v}_{тест}\times\vec{B}.$$

    Направление силы определяется правилом правой руки и перпендикулярно плоскости, определяемой вектором скорости и магнитным полем. Это означает, что заряд не будет испытывать магнитной силы, если он движется прямо вдоль линии магнитного поля (или против нее).

    Сбор движущихся зарядов

    Магнитное поле от линии заряда

    Одиночный движущийся заряд, конечно, не единственная ситуация, с которой мы сталкиваемся. На самом деле довольно часто группа зарядов движется, образуя ток. Этот набор движущихся зарядов или настоящий ток также порождает магнитное поле, и, подобно электрическому полю, магнитное поле подчиняется суперпозиции. Основная предпосылка очень похожа на электрическое поле,

    $$\vec{B}_{net} = \vec{B}_1 + \vec{B}_2 + \vec{B}_3 + \dots = \sum_i \vec{B}_i$$

    Этот результат говорит нам о том, как работает общее магнитное поле из-за распределения движущихся зарядов, но как насчет истинного тока ($I$)? Мы можем относиться к каждому маленькому сегменту провода таким образом, чтобы задать вопрос: какой вклад вы вносите в магнитное поле? Когда мы делаем это, мы получаем небольшой вклад от каждого сегмента (длина $dl$) и суммируем их, используя интеграл,

    $$d\vec{B} = \dfrac{\mu_0}{4\pi} \dfrac{I d\vec{l} \times \hat{r}}{r^2}$$ 92}$$

    Мы можем использовать этот метод интегрирования аналитически и вычислительно, чтобы найти магнитное поле, создаваемое распределением тока — либо через отрезок провода, либо через очень длинный провод, либо через петли проводов.

    Эффекты и приложения

    Тот факт, что движущиеся заряды генерируют магнитные поля, что они накладываются друг на друга, и что другие движущиеся заряды испытывают магнитные силы в присутствии магнитного поля, приводят к целому ряду различных магнитных явлений. Некоторые вполне практичны. Некоторые из наиболее важных из них обсуждаются ниже:

    Магнитное поле вокруг провода с током

    Провода с током

    Провода, по которым течет ток, создают магнитные поля, которые циркулируют вокруг них. Это происходит из-за наложения магнитных полей от всех маленьких зарядов, движущихся по проводу. Это может быть очень важно, потому что эти магнитные поля могут мешать другой чувствительной электронике в конкретном электронном устройстве или эксперименте. Более того, пары токонесущих проводов могут воздействовать друг на друга силами (притягивающими или отталкивающими), вызывая физическую нагрузку на провода или другие компоненты.

    Криволинейное движение

    Движение заряда в магнитном поле

    Магнитные поля не могут изменить кинетическую энергию заряженных частиц. Это связано с тем, что магнитная сила, действующая на частицу, всегда перпендикулярна движению частицы. Так магнитные поля могут изменить траекторию частицы, но не способны ускорить или замедлить движение частицы. В результате частица, движущаяся в однородном магнитном поле без каких-либо других сил, будет совершать равномерное круговое движение. Направление орбиты (по часовой или против часовой стрелки) будет зависеть от знака заряда, направления скорости и направления магнитного поля.

    Это очень полезный результат для циклотронных ускорителей, где магнитное поле используется для того, чтобы заставить заряженные частицы двигаться по круговой траектории (в то время как электрическое поле используется для ускорения заряженных частиц). Круговой путь (созданный магнитным полем) помогает удерживать ускоритель на меньшей площади. Мы также можем использовать тенденцию к искривлению траекторий в масс-спектрометрах, которые можно использовать для определения отношения заряда к массе частицы, поскольку частицы с разным отношением заряда к массе будут изгибаться по-разному. Используя совместно электрические и магнитные поля, мы можем манипулировать траекториями заряженных частиц таким образом, чтобы отбирать только определенные виды частиц; это используется в ускорителях частиц, а также в более простых ситуациях, таких как электронно-лучевые экраны (которые раньше были в больших квадратных телевизорах и компьютерных мониторах, но в последнее время вышли из моды).

    Изменение магнитных полей

    Большая часть нашего исследования магнитных полей была сосредоточена на полях, обусловленных постоянными токами, то есть на магнитных полях, которые не изменялись со временем. Поле создавалось постоянным потоком носителей заряда, которые мы моделировали как движущиеся с одинаковой скоростью и направлением. Это не обязательно должно быть так, и довольно часто это не так.

    В случае тока, изменяющегося во времени, мы создаем изменяющееся во времени магнитное поле. Наблюдается, что этот вид магнитного поля генерирует искривленное электрическое поле.

alexxlab

Добавить комментарий

Ваш адрес email не будет опубликован. Обязательные поля помечены *